CAT Verbal Ability Questions | CAT Para Jumbles questions

Every year 3 to 4 questions are asked from Para jumbles in VARC Section. To practise CAT past year questions on Para Jumbles, click here. Best explanation and authentic Actual CAT Answer key

CAT/2023.3(Verbal Ability)

Question. 1

The four sentences (labelled 1, 2, 3 and 4) given below, when properly sequenced, would yield a coherent paragraph. Decide on the proper sequencing of the order of the sentences and key in the sequence of the four numbers as your answer.

1. Centuries later formal learning is still mostly based on reading, even with the widespread use of other possible education-affecting technologies such as film, radio, and television.

2. One of the immediate and recognisable impacts of the printing press was on how people learned; in the scribal culture it primarily involved listening, so memorization was paramount.

3. The transformation of learners from listeners to readers was a complex social and cultural phenomenon, and it was not until the industrial era that the concept of universal literacy took root.

4. The printing press shifted the learning process, as listening and memorisation gradually gave way to reading and learning no longer required the presence of a mentor; it could be done privately.

Explanation

CAT/2023.3(Verbal Ability)

Question. 2

The four sentences (labelled 1, 2, 3 and 4) given below, when properly sequenced, would yield a coherent paragraph. Decide on the proper sequencing of the order of the sentences and key in the sequence of the four numbers as your answer.

1. Veena Sahajwalla, a materials scientist at the University of New South Wales, believes there is a new way of solving this problem.

2. Her vision is for automated drones and robots to pick out components, put them into a small furnace and smelt them at specific temperatures to extract the metals one by one before they are sent off to manufacturers for reuse.

3. E-waste contains huge quantities of valuable metals, ceramics and plastics that could be salvaged and recycled, although currently not enough of it is.

4. She plans to build microfactories that can tease apart the tangle of materials in mobile phones, computers and other e-waste.

Explanation

CAT/2023.2(Verbal Ability)

Question. 3

The four sentences (labelled 1, 2, 3 and 4) given below, when properly sequenced, would yield a coherent paragraph. Decide on the proper sequencing of the order of the sentences and key in the sequence of the four numbers as your answer.

1. Like the ants that make up a colony, no single neuron holds complex information like self-awareness, hope or pride.

2. Although the human brain is not yet understood enough to identify the mechanism by which emergence functions, most neurobiologists agree that complex interconnections among the parts give rise to qualities that belong only to the whole.

3. Nonetheless, the sum of all neurons in the nervous system generate complex human emotions like fear and joy, none of which can be attributed to a single neuron.

4. Human consciousness is often called an emergent property of the human brain.

Explanation

CAT/2023.2(Verbal Ability)

Question. 4

The four sentences (labelled 1, 2, 3 and 4) given below, when properly sequenced, would yield a coherent paragraph. Decide on the proper sequencing of the order of the sentences and key in the sequence of the four numbers as your answer.

1. Contemporary African writing like ‘The Bottled Leopard’ voices this theme using two children and two backgrounds to juxtapose two varying cultures.

2. Chukwuemeka Ike explores the conflict, and casts the Western tradition as condescending, enveloping and unaccommodating towards local African practice.

3. However, their views contradict the reality, for a rich and sustaining local African cultural ethos exists for all who care, to see and experience.

4. Western Christian concepts tend to deny or feign ignorance about the existence of a genuine and enduring indigenous African tradition

Explanation

CAT/2023.1(Verbal Ability)

Question. 5

The four sentences (labelled 1, 2, 3, 4) below, when properly sequenced would yield a coherent paragraph. Decide on the proper sequencing of the order of the sentences and key in the sequence of the four numbers as your answer:

1. What precisely are the “unusual elements” that make a particular case so attractive to a certain kind of audience?

2 . It might be a particularly savage or unfathomable level of depravity, very often it has something to do with the precise amount of mystery involved.

3. Unsolved, and perhaps unsolvable cases offer something that “ordinary” murder doesn’t.

4. Why are some crimes destined for perpetual re-examination and others locked into permanent obscurity?

Explanation

CAT/2023.1(Verbal Ability)

Question. 6

The four sentences (labelled 1, 2, 3 and 4) given below, when properly sequenced, would yield a coherent paragraph. Decide on the proper sequencing of the order of the sentences and key in the sequence of the four numbers as your answer.

1. Algorithms hosted on the internet are accessed by many, so biases in AI models have resulted in much larger impact, adversely affecting far larger groups of people.

2. Though “algorithmic bias” is the popular term, the foundation of such bias is not in algorithms, but in the data; algorithms are not biased, data is, as algorithms merely reflect persistent patterns that are present in the training data.

3. Despite their widespread impact, it is relatively easier to fix AI biases than human-generated biases, as it is simpler to identify the former than to try to make people unlearn behaviors learnt over generations.

4. The impact of biased decisions made by humans is localised and geographically confined, but with the advent of AI, the impact of such decisions is spread over a much wider scale.

Explanation

CAT/2022.3(Verbal Ability)

Question. 7

The four sentences (labelled 1, 2, 3 and 4) below, when properly sequenced, would yield a coherent paragraph. Decide on the proper sequencing of the order of the sentences and key in the sequence of the four numbers as your answer:

1.Various industrial sectors including retail, transit systems, enterprises, educational institutions, event organizing, finance, travel etc. have now started leveraging these beacons solutions to track and communicate with their customers.

2.A beacon fixed on to a shop wall enables the retailer to assess the proximity of the customer, and come up with a much targeted or personalized communication like offers, discounts and combos on products in each shelf.

3.Smart phones or other mobile devices can capture the beacon signals, and distance can be estimated by measuring received signal strength.

4.Beacons are tiny and inexpensive, micro-location-based technology devices that can send radio frequency signals and notify nearby Bluetooth devices of their presence and transmit information.

Explanation

CAT/2022.3(Verbal Ability)

Question. 8

The four sentences (labelled 1, 2, 3 and 4) below, when properly sequenced, would yield a coherent paragraph. Decide on the proper sequencing of the order of the sentences and key in the sequence of the four numbers as your answer:

1. If I wanted to sit indoors and read, or play Sonic the Hedgehog on a red-hot Sega Mega Drive, I would often be made to feel guilty about not going outside to “enjoy it while it lasts”.

2. My mum, quite reasonably, wanted me and my sister out of the house, in the sun.

3. Tales of my mum’s idyllic-sounding childhood in the Sussex countryside, where trees were climbed by 8 am and streams navigated by lunchtime, were passed down to us like folklore.

4. To an introverted kid, that felt like a threat – and the feeling has stayed with me.

Explanation

CAT/2022.3(Verbal Ability)

Question. 9

The four sentences (labelled 1, 2, 3 and 4) below, when properly sequenced, would yield a coherent paragraph. Decide on the proper sequencing of the order of the sentences and key in the sequence of the four numbers as your answer:

1. The more we are able to accept that our achievements are largely out of our control, the easier it becomes to understand that our failures, and those of others, are too.

2. But the raft of recent books about the limits of merit is an important correction to the arrogance of contemporary entitlement and an opportunity to reassert the importance of luck, or grace, in our thinking.

3. Meritocracy as an organising principle is an inevitable function of a free society, as we are designed to see our achievements as worthy of reward.

4. And that in turn should increase our humility and the respect with which we treat our fellow citizens, helping ultimately to build a more compassionate society.

Explanation

CAT/2022.2(Verbal Ability)

Question. 10

 The four sentences (labelled 1, 2, 3 and 4) below, when properly sequenced, would yield a coherent paragraph. Decide on the proper sequencing of the order of the sentences and key in the sequence of the four numbers as your answer:

1. From chemical pollutants in the environment to the damming of rivers to invasive species transported through global trade and travel, every environmental issue is different and there is no single tech solution that can solve this crisis.

2. Discourse on the threat of environmental collapse revolves around cutting down emissions, but biodiversity loss and ecosystem collapse are caused by myriad and diverse reasons.

3. This would require legislation that recognises the rights of future generations and other species that allows the judiciary to uphold a much higher standard of environmental protection than currently possible.

4. Clearly, our environmental crisis requires large political solutions, not minor technological ones, so, instead of focusing on infinite growth, we could consider a path of stable-state economies, while preserving markets and healthy competition.

Explanation

CAT/2022.2(Verbal Ability)

Question. 11

 The four sentences (labelled 1, 2, 3 and 4) below, when properly sequenced, would yield a coherent paragraph. Decide on the proper sequencing of the order of the sentences and key in the sequence of the four numbers as your answer:

1. Women may prioritize cooking because they feel they alone are responsible for mediating a toxic and unhealthy food system.

2. Food is commonly framed through the lens of individual choice: you can choose to eat healthily.

3. This is particularly so in a neoliberal context where the state has transferred the responsibility for food onto individual consumers.

4. The individualized framing of choice appeals to a popular desire to experience agency, but draws away from the structural obstacles that stratify individual food choices.

Explanation

CAT/2022.2(Verbal Ability)

Question. 12

 The four sentences (labelled 1, 2, 3 and 4) below, when properly sequenced, would yield a coherent paragraph. Decide on the proper sequencing of the order of the sentences and key in the sequence of the four numbers as your answer:

1. The trajectory of cheerfulness through the self is linked to the history of the word ‘cheer’ which comes from an Old French meaning ‘face’.

2. Translations of the Bible into vernacular languages, expanded the noun ‘cheer’ into the more abstract ‘cheerful-ness’, something that circulates as an emotional and social quality defining the self and a moral community.

3. When you take on a cheerful expression, no matter what the state of your soul, your cheerfulness moves into the self: the interior of the self is changed by the power of cheer.

4. People in the medieval ‘Canterbury Tales’ have a ‘piteous’ or a ‘sober’ cheer; ‘cheer’ is an expression and a body part, lying at the intersection of emotions and physiognomy.

Explanation

CAT/2022.1(Verbal Ability)

Question. 13

The four sentences (labelled 1, 2, 3 and 4) below, when properly sequenced, would yield a coherent paragraph. Decide on the proper sequencing of the order of the sentences and key in the sequence of the four numbers as your answer:

1. Some company leaders are basing their decisions on locating offices to foster innovation and growth, as their best-performing inventors suffered the greatest productivity losses when their commutes grew longer.

2. Shorter commutes support innovation by giving employees more time in the office and greater opportunities for in-person collaboration, while removing the physical strain of a long commute.

3. This is not always the case: remote work does not automatically lead to greater creativity and productivity as office water-cooler conversations are also very important for innovation.

4. Some see the link between long commutes and productivity as support for workfrom-home scenarios, as many workers have grown accustomed to their commutefree arrangements during the pandemic.

Explanation

CAT/2022.1(Verbal Ability)

Question. 14

The four sentences (labelled 1, 2, 3 and 4) below, when properly sequenced, would yield a coherent paragraph. Decide on the proper sequencing of the order of the sentences and key in the sequence of the four numbers as your answer:

1. The creative element in product design has become of paramount importance as it is one of the few ways a firm or industry can sustain a competitive advantage over its rivals.

2. In fact, the creative element in the value of world industry would be larger still, if we added the contribution of the creative element in other industries, such as the design of tech accessories.

3. The creative industry is receiving a lot of attention today as its growth rate is faster than that of the world economy as a whole.

4. It is for this reason that today’s trade issues are increasingly involving intellectual property, as Western countries have an interest in protecting their revenues along with freeing trade in non-tangibles.

Explanation

CAT/2022.1(Verbal Ability)

Question. 15

The four sentences (labelled 1, 2, 3 and 4) below, when properly sequenced, would yield a coherent paragraph. Decide on the proper sequencing of the order of the sentences and key in the sequence of the four numbers as your answer:

1. Fish skin collagen has excellent thermo-stability and tensile strength making it ideal for use as bandage that adheres to the skin and adjusts to body movements.

2. Collagen, one of the main structural proteins in connective tissues in the human body, is well known for promoting skin regeneration.

3. Fish skin swims in here as diseases and bacteria that affect fish are different from most human pathogens.

4. The risk of introducing disease agents into other species through the use of pig and cow collagen proteins for wound healing has inhibited its broader applications in the medical field.

Explanation

CAT/2021.1(Verbal Ability)

Question. 16

The four sentences (labelled 1, 2, 3, 4) below, when properly sequenced would yield a coherent paragraph. Decide on the proper sequencing of the order of the sentences and key in the sequence of the four numbers as your answer:

  1. The work is more than the text, for the text only takes on life, when it is realized and furthermore the realization is by no means independent of the individual disposition of the reader.
  2. The convergence of text and reader brings the literary work into existence and this convergence is not to be identified either with the reality of the text or with the individual disposition of the reader.
  3. From this polarity it follows that the literary work cannot be completely identical with the text, or with the realization of the text, but in fact must lie halfway between the two.
  4. The literary work has two poles, which we might call the artistic and the aesthetic; the artistic refers to the text created by the author, and the aesthetic to the realization accomplished by the reader.

Explanation

CAT/2021.1(Verbal Ability)

Question. 17

Five jumbled up sentences, related to a topic, are given below. Four of them can be put together to form a coherent paragraph. Identify the odd one out and key in the number of the sentence as your answer:

1. The legal status of resources mined in space remains ambiguous; and while the market for asteroid minerals is currently nonexistent, this is likely to change as technical hurdles diminish.

2. Outer space is a commons, and all of it is open for exploration, however, space law developed in the 1950s and 60s is state-centric and arguably ill-suited to a commercial future.

3. Laws adopted by the US and Luxembourg are first steps, but they only protect firms from competing claims by their compatriots; a Chinese company will not be bound by US law.

4. Critics say the US is conferring rights that it has no authority to confer; Russia in particular has condemned this, citing the US’ disrespect for international law.

5. At issue now is commercial activity, as private firms—rather than nation states—look to space for profit.

Explanation

CAT/2021.2(Verbal Ability)

Question. 18

The four sentences (labelled 1, 2, 3, 4) below, when properly sequenced would yield a coherent paragraph. Decide on the proper sequencing of the order of the sentences and key in the sequence of the four numbers as your answer:

  1. But today there is an epochal challenge to rethink and reconstitute the vision and practice of development as a shared responsibility – a sharing which binds both the agent and the audience, the developed world and the developing, in a bond of shared destiny.
  2. We are at a crossroads now in our vision and practice of development.
  3. This calls for the cultivation of an appropriate ethical mode of being in our lives which enables us to realize this global and planetary situation of shared living and responsibility.
  4. Half a century ago, development began as a hope for a better human possibility, but in the last fifty years, this hope has lost itself in the dreary desert of various kinds of hegemonic applications.

Explanation

CAT/2021.2(Verbal Ability)

Question. 19

The four sentences (labelled 1, 2, 3 and 4) below, when properly sequenced would yield a coherent paragraph. Decide on the proper sequencing of the order of the sentences and key in the sequence of the four numbers as your answer:

  1. Look forward a few decades to an invention which can end the energy crisis, change the global economy and curb climate change at a stroke: commercial fusion power.
  2. To gain meaningful insights, logic has to be accompanied by asking probing questions of nature through controlled tests, precise observations and clever analysis.
  3. The greatest of all inventions is the über-invention that has provided the insights on which others depend: the modern scientific method.
  4. This invention is inconceivable without the scientific method; it will rest on the application of a diverse range of scientific insights, such as the process transforming hydrogen into helium to release huge amounts of energy.

Explanation

CAT/2021.2(Verbal Ability)

Question. 20

The four sentences (labelled 1, 2, 3 and 4) below, when properly sequenced would yield a coherent paragraph. Decide on the proper sequencing of the order of the sentences and key in the sequence of the four numbers as your answer:

  1. The US has long maintained that the Northwest Passage is an international strait through which its commercial and military vessels have the right to pass without seeking Canada’s permission.
  2. Canada, which officially acquired the group of islands forming the Northwest Passage in 1880, claims sovereignty over all the shipping routes through the Passage.
  3. The dispute could be transitory, however, as scientists speculate that the entire Arctic Ocean will soon be ice-free in summer, so ship owners will not have to ask for permission to sail through any of the Northwest Passage routes.
  4. The US and Canada have never legally settled the question of access through the Passage, but have an agreement whereby the US needs to seek Canada’s consent for any transit.

Explanation

CAT/2021.3(Verbal Ability)

Question. 21

The four sentences (labelled 1, 2, 3, and 4) below, when properly sequenced would yield a coherent paragraph. Decide on the proper sequencing of the order of the sentences and key in the sequence of the four numbers as your answer:

  1. Businesses find automation, such as robotic employees, a big asset in terms of productivity and efficiency.
  2. But in recent years, robotics has had increasing impacts on unemployment, not just of manual labour, as computers are rapidly handling some white-collar and service-sector work.
  3. For years politicians have promised workers that they would bring back their jobs by clamping down on trade, offshoring, and immigration.
  4. Economists, based on their research, say that the bigger threat to jobs now is not globalization but automation.

Explanation

CAT/2021.3(Verbal Ability)

Question. 22

The four sentences (labelled 1, 2, 3 and 4) below, when properly sequenced would yield a coherent paragraph. Decide on the proper sequencing of the order of the sentences and key in the sequence of the four numbers as your answer:

  1. Restitution of artefacts to original cultures could faces legal obstacles, as many Western museums are legally prohibited from disposing off their collections.
  2. This is in response to countries like Nigeria, which are pressurising European museums to return their precious artefacts looted by colonisers in the past.
  3. Museums in Europe today are struggling to come to terms with their colonial legacy, some taking steps to return artefacts but not wanting to lose their prized collections.
  4. Legal hurdles notwithstanding, politicians and institutions in France and Germany would now like to defuse the colonial time bombs, and are now backing the return of part of their holdings.

Explanation

CAT/2021.3(Verbal Ability)

Question. 23

The four sentences (labelled 1, 2, 3, 4) below, when properly sequenced would yield a coherent paragraph. Decide on the proper sequencing of the order of the sentences and key in the sequence of the four numbers as your answer:

  1. It is regimes of truth that make certain relationships speakable – relationships, like subjectivities, are constituted through discursive formations, which sustain regimes of truth.
  2. Relationships are nothing without the communication that brings them into being; interpersonal communication is connected to knowledge shared by interlocutors, and scholars should attend to relational histories in their analyses.
  3. A Foucauldian approach to relationships goes beyond these conceptions of discourse and history to macrolevel regimes of truth as constituting relationships.
  4. Reconsidering micropractices within relationships that are constituted within and simultaneously contributors to regimes of truth acknowledges the central position of power/knowledge in the constitution of what has come to be considered true and real.

Explanation

CAT/2020.1(Verbal Ability)

Question. 24

The four sentences (labelled 1, 2, 3, 4) below, when properly sequenced would yield a coherent paragraph. Decide on the proper sequencing of the order of the sentences and key in the sequence of the four numbers as your answer:

1. Tensions and sometimes conflict remain an issue in and between the 11 states in South East Asia (Brunei Darussalam, Cambodia, Indonesia, Laos, Malaysia, Myanmar, the Philippines, Singapore, Thailand, Timor-Leste and Vietnam).
2. China’s rise as a regional military power and its claims in the South China Sea have become an increasingly pressing security concern for many South East Asian states.
3. Since the 1990s, the security environment of South East Asia has seen both continuity and profound changes.
4. These concerns cause states from outside the region to take an active interest in South East Asian security.

Explanation

CAT/2020.1(Verbal Ability)

Question. 25

The four sentences (labelled 1, 2, 3, 4) below, when properly sequenced would yield a coherent paragraph. Decide on the proper sequencing of the order of the sentences and key in the sequence of the four numbers as your answer:

1. Relying on narrative structure alone, indigenous significances of nineteenth century San folktales are hard to determine.
2. Using their supernatural potency, benign shamans transcend the levels of the San cosmos in order to deal with social conflict and to protect material resources and enjoy a measure of respect that sets them apart from ordinary people.
3. Selected tales reveal that they deal with a form of spiritual conflict that has social implications and concern conflict between people and living or dead malevolent shamans.
4. Meaning can be elicited, and the tales contextualized, by probing beneath the narrative of verbatim, original-language records and exploring the connotations of highly significant words and phrases.

Explanation

CAT/2020.1(Verbal Ability)

Question. 26

The four sentences (labelled 1, 2, 3, 4) below, when properly sequenced would yield a coherent paragraph. Decide on the proper sequencing of the order of the sentences and key in the sequence of the four numbers as your answer:

1. Man has used poisons for assassination purposes ever since the dawn of civilization, against individual enemies but also occasionally against armies.
2. These dangers were soon recognized, and resulted in two international declarations—in 1874 in Brussels and in 1899 in The Hague—that prohibited the use of poisoned weapons.
3. The foundation of microbiology by Louis Pasteur and Robert Koch offered new prospects for those interested in biological weapons because it allowed agents to be chosen and designed on a rational basis.
4. Though treaties were all made in good faith, they contained no means of control, and so failed to prevent interested parties from developing and using biological weapons.

Explanation

CAT/2020.2(Verbal Ability)

Question. 27

The four sentences (labelled 1, 2, 3, 4) below, when properly sequenced would yield a coherent paragraph. Decide on the proper sequencing of the order of the sentences and key in the sequence of the four numbers as your answer:

1. But the attention of the layman, not surprisingly, has been captured by the atom bomb, although there is at least a chance that it may never be used again.
2. Of all the changes introduced by man into the household of nature, [controlled] large-scale nuclear fission is undoubtedly the most dangerous and most profound.
3. The danger to humanity created by the so-called peaceful uses of atomic energy may, however, be much greater.
4. The resultant ionizing radiation has become the most serious agent of pollution of the environment and the greatest threat to man’s survival on earth.

Explanation

CAT/2020.2(Verbal Ability)

Question. 28

The four sentences (labelled 1, 2, 3, 4) below, when properly sequenced would yield a coherent paragraph. Decide on the proper sequencing of the order of the sentences and key in the sequence of the four numbers as your answer:

1. While you might think that you see or are aware of all the changes that happen in your immediate environment, there is simply too much information for your brain to fully process everything.
2. Psychologists use the term ‘change blindness’ to describe this tendency of people to be blind to changes though they are in the immediate environment.
3. It cannot be aware of every single thing that happens in the world around you.
4. Sometimes big shifts happen in front of your eyes and you are not at all aware of these changes.

Explanation

CAT/2020.2(Verbal Ability)

Question. 29

The four sentences (labelled 1, 2, 3, 4) below, when properly sequenced would yield a coherent paragraph. Decide on the proper sequencing of the order of the sentences and key in the sequence of the four numbers as your answer:

1. It also has four movable auxiliary telescopes 1.8 m in diameter.
2. Completed in 2006, the Very Large Telescope (VLT) has four reflecting telescopes, 8.2 m in diameter that can observe objects 4 billion times weaker than can normally be seen with the naked eye.
3. This configuration enables one to distinguish an astronaut on the Moon.
4. When these are combined with the large telescopes, they produce what is called interferometry: a simulation of the power of a mirror 16 m in diameter and the resolution of a telescope of 200 m.

Explanation

CAT/2020.3(Verbal Ability)

Question. 30

The four sentences (labelled 1, 2, 3, 4) below, when properly sequenced would yield a coherent paragraph. Decide on the proper sequencing of the order of the sentences and key in the sequence of the four numbers as your answer:

1. It advocated a conservative approach to antitrust enforcement that espouses faith in efficient markets and voiced suspicion regarding the merits of judicial intervention to correct anti-competitive practices.
2. Many industries have consistently gained market share, the lion’s share - without any official concern; the most successful technology companies have grown into veritable titans, on the premise that they advance ‘public interest’.
3. That the new anticompetitive risks posed by tech giants like Google, Facebook, and Amazon, necessitate new legal solutions could be attributed to the dearth of enforcement actions against monopolies and the few cases challenging mergers in the USA.
4. The criterion of ‘consumer welfare standard’ and the principle that antitrust law should serve consumer interests and that it should protect competition rather than individual competitors was an antitrust law introduced by, and named after, the 'Chicago school'. 

 

Explanation

CAT/2020.3(Verbal Ability)

Question. 31

The four sentences (labelled 1, 2, 3, 4) below, when properly sequenced would yield a coherent paragraph. Decide on the proper sequencing of the order of the sentences and key in the sequence of the four numbers as your answer:

1. Each one personified a different aspect of good fortune.
2. The others were versions of popular Buddhist gods, Hindu gods and Daoist gods.
3. Seven popular Japanese deities, the Shichi Fukujin, were considered to bring good luck and happiness.
4. Although they were included in the Shinto pantheon, only two of them, Daikoku and Ebisu, were indigenous Japanese gods.

Explanation

CAT/2020.3(Verbal Ability)

Question. 32

The four sentences (labelled 1, 2, 3, 4) below, when properly sequenced would yield a coherent paragraph. Decide on the proper sequencing of the order of the sentences and key in the sequence of the four numbers as your answer:

1. Complex computational elements of the CNS are organized according to a “nested” hierarchic criterion; the organization is not permanent and can change dynamically from moment to moment as they carry out a computational task.
2. Echolocation in bats exemplifies adaptation produced by natural selection; a function not produced by natural selection for its current use is exaptation -- feathers might have originally arisen in the context of selection for insulation.
3. From a structural standpoint, consistent with exaptation, the living organism is organized as a complex of “Russian Matryoshka Dolls” -- smaller structures are contained within larger ones in multiple layers.
4. The exaptation concept, and the Russian-doll organization concept of living beings deduced from studies on evolution of the various apparatuses in mammals, can be applied for the most complex human organ: the central nervous system (CNS).

Explanation

CAT/2019.1(Verbal Ability)

Question. 33

The four sentences (labelled 1, 2, 3, 4) given below, when properly sequenced would yield a coherent paragraph. Decide on the proper sequence of the order of the sentences and key in the sequence of the four numbers as your answer.

1. People with dyslexia have difficulty with print-reading, and people with autism spectrum disorder have difficulty with mind-reading.
2. An example of a lost cognitive instinct is mind-reading: our capacity to think of ourselves and others as having beliefs, desires, thoughts and feelings.
3. Mind-reading looks increasingly like literacy, a skill we know for sure is not in our genes, since scripts have been around for only 5,000-6,000 years.
4. Print-reading, like mind-reading varies across cultures, depends heavily on certain parts of the brain, and is subject to developmental disorders.

Explanation

CAT/2019.1(Verbal Ability)

Question. 34

The four sentences (labelled 1, 2, 3, 4) given below, when properly sequenced would yield a coherent paragraph. Decide on the proper sequence of the order of the sentences and key in the sequence of the four numbers as your answer.

1. If you’ve seen a little line of text on websites that says something like "customers who bought this also enjoyed that” you have experienced this collaborative filtering firsthand.
2. The problem with these algorithms is that they don’t take into account a host of nuances and circumstances that might interfere with their accuracy.
3. If you just bought a gardening book for your cousin, you might get a flurry of links to books about gardening, recommended just for you! – the algorithm has no way of knowing you hate gardening and only bought the book as a gift.
4. Collaborative filtering is a mathematical algorithm by which correlations and cooccurrences of behaviors are tracked and then used to make recommendations.

Explanation

CAT/2019.1(Verbal Ability)

Question. 35

The four sentences (labelled 1, 2, 3, 4) given below, when properly sequenced would yield a coherent paragraph. Decide on the proper sequence of the order of the sentences and key in the sequence of the four numbers as your answer.

1. We’ll all live under mob rule until then, which doesn’t help anyone.
2. Perhaps we need to learn to condense the feedback we receive online so that 100 replies carry the same weight as just one.
3. As we grow more comfortable with social media conversations being part of the way we interact every day, we are going to have to learn how to deal with legitimate criticism.
4. A new norm will arise where it is considered unacceptable to reply with the same point that dozens of others have already.

Explanation

CAT/2019.1(Verbal Ability)

Question. 36

The four sentences (labelled 1, 2, 3, 4) given below, when properly sequenced would yield a coherent paragraph. Decide on the proper sequence of the order of the sentences and key in the sequence of the four numbers as your answer.

1. Metaphors may map to similar meanings across languages, but their subtle differences can have a profound effect on our understanding of the world.
2. Latin scholars point out carpe diem is a horticultural metaphor that, particularly seen in the context of its source, is more accurately translated as “plucking the day,” evoking the plucking and gathering of ripening fruits or flowers, enjoying a moment that is rooted in the sensory experience of nature, unrelated to the force implied in seizing.
3. The phrase carpe diem, which is often translated as “seize the day and its accompanying philosophy, has gone on to inspire countless people in how they live their lives and motivates us to see the world a little differently from the norm.
4. It’s an example of one of the more telling ways that we mistranslate metaphors from one language to another, revealing in the process our hidden assumptions about what we really value.

Explanation

CAT/2019.2(Verbal Ability)

Question. 37

The four sentences (labelled 1, 2, 3, 4) given below, when properly sequenced would yield a coherent paragraph. Decide on the proper sequence of the order of the sentences and key in the sequence of the four numbers as your answer.

1. Conceptualisations of ‘women’s time’ as contrary to clock-time and clock-time as synonymous with economic rationalism are two of the deleterious results of this representation.
2. While dichotomies of ‘men’s time’, ‘women’s time’, clock-time, and caring time can be analytically useful, this article argues that everyday caring practices incorporate a multiplicity of times; and both men and women can engage in these multiple-times
3. When the everyday practices of working sole fathers and working sole mothers are carefully examined to explore conceptualisations of gendered time, it is found that caring time is often more focused on the clock than generally theorised.
4. Clock-time has been consistently represented in feminist literature as a masculine artefact representative of a ‘time is money’ perspective.

Explanation

CAT/2019.2(Verbal Ability)

Question. 38

The four sentences (labelled 1, 2, 3, 4) given below, when properly sequenced would yield a coherent paragraph. Decide on the proper sequence of the order of the sentences and key in the sequence of the four numbers as your answer.

1. Living things—animals and plants—typically exhibit correlational structure.
2. Adaptive behaviour depends on cognitive economy, treating objects as equivalent.
3. The information we receive from our senses, from the world, typically has structure and order, and is not arbitrary.
4. To categorize an object means to consider it equivalent to other things in that category, and different—along some salient dimension—from things that are not.

Explanation

CAT/2019.2(Verbal Ability)

Question. 39

The four sentences (labelled 1, 2, 3, 4) given below, when properly sequenced would yield a coherent paragraph. Decide on the proper sequence of the order of the sentences and key in the sequence of the four numbers as your answer.

1. To the uninitiated listener, atonal music can sound like chaotic, random noise.
2. Atonality is a condition of music in which the constructs of the music do not ‘live’ within the confines of a particular key signature, scale, or mode.
3. After you realize the amount of knowledge, skill, and technical expertise required to compose or perform it, your tune may change, so to speak.
4. However, atonality is one of the most important movements in 20th century music.

Explanation

CAT/2019.2(Verbal Ability)

Question. 40

The four sentences (labelled 1, 2, 3, 4) given below, when properly sequenced would yield a coherent paragraph. Decide on the proper sequence of the order of the sentences and key in the sequence of the four numbers as your answer.

1. Such a belief in the harmony of nature requires a purpose presumably imposed by the goodness and wisdom of a deity.
2. These parts, all fit together into an integrated, well-ordered system that was created by design.
3. Historically, the notion of a balance of nature is part observational, part metaphysical, and not scientific in any way.
4. It is an example of an ancient belief system called teleology, the notion that what we call nature has a predetermined destiny associated with its component parts.

Explanation

CAT/2018.1(Verbal Ability)

Question. 41

The four sentences (labelled 1, 2, 3, 4) given below, when properly sequenced would yield a coherent paragraph. Decide on the proper sequence of the order of the sentences and key in the sequence of the four numbers as your answer.

1. Impartiality and objectivity are fiendishly difficult concepts that can cause all sorts of injustices even if transparently implemented.
2. It encourages us into bubbles of people we know and like, while blinding us to different perspectives, but the deeper problem of ‘transparency’ lies in the words “and much more”.
3. Twitter’s website says that “tweets you are likely to care about most will show up first in your timeline based on accounts you interact with most, tweets you engage with, and much more.”
4. We are only told some of the basic principles, and we can’t see the algorithm itself, making it hard for citizens to analyse the system sensibly or fairly or be convinced of its impartiality and objectivity.

Explanation

CAT/2018.1(Verbal Ability)

Question. 42

The four sentences (labelled 1, 2, 3, 4) given below, when properly sequenced would yield a coherent paragraph. Decide on the proper sequence of the order of the sentences and key in the sequence of the four numbers as your answer.

1. The eventual diagnosis was skin cancer and after treatment all seemed well.
2. The viola player didn’t know what it was; nor did her GP
3. Then a routine scan showed it had come back and spread to her lungs.
4. It started with a lump on Cathy Perkins’ index finger.

Explanation

CAT/2018.1(Verbal Ability)

Question. 43

The four sentences (labelled 1, 2, 3, 4) given below, when properly sequenced would yield a coherent paragraph. Decide on the proper sequence of the order of the sentences and key in the sequence of the four numbers as your answer.

1. The woodland’s canopy receives most of the sunlight that falls on the trees.
2. Swifts do not confine themselves to woodlands, but hunt wherever there are insects in the air.
3. With their streamlined bodies, swifts are agile flyers, ideally adapted to twisting and turning through the air as they chase flying insects – the creatures that form their staple diet.
4. Hundreds of thousands of insects fly in the sunshine up above the canopy, some falling prey to swifts and swallows.

Explanation

CAT/2018.1(Verbal Ability)

Question. 44

The four sentences (labelled 1, 2, 3, 4) given below, when properly sequenced would yield a coherent paragraph. Decide on the proper sequence of the order of the sentences and key in the sequence of the four numbers as your answer.

1. But now we have another group: the unwitting enablers.
2. Democracy and high levels of inequality of the kind that have come to characterize the United States are simply incompatible.
3. Believing these people are working for a better world, they are, actually, at most, chipping away at the margins, making slight course corrections, ensuring the system goes on as it is, uninterrupted.
4. Very rich people will always use money to maintain their political and economic power.

Explanation

CAT/2018.2(Verbal Ability)

Question. 45

The four sentences (labelled 1, 2, 3, 4) given below, when properly sequenced would yield a coherent paragraph. Decide on the proper sequence of the order of the sentences and key in the sequence of the four numbers as your answer.

1. It was his taxpayers who had to shell out as much as $1.6bn over 10 years to employees of failed companies.
2. Companies in many countries routinely engage in such activities which means that the employees are left with unpaid entitlements.
3. Deliberate and systematic liquidation of a company to avoid liabilities and then restarting the business is called phoenixing.
4. The Australian Minister for Revenue and Services discovered in an audit that phoenixing had cost the Australian economy between 2.9bn and 2.9bn and 5.1bn last year.

Explanation

CAT/2018.2(Verbal Ability)

Question. 46

The four sentences (labelled 1, 2, 3, 4) given below, when properly sequenced would yield a coherent paragraph. Decide on the proper sequence of the order of the sentences and key in the sequence of the four numbers as your answer.

1. Self-management is thus defined as the ‘individual’s ability to manage the symptoms, treatment, physical and psychosocial consequences and lifestyle changes inherent in living with a chronic condition’.
2. Most people with progressive diseases like dementia prefer to have control over their own lives and health-care for as long as possible.
3. Having control means, among other things, that patients themselves perform self-management activities.
4. Supporting people in decisions and actions that promote self-management is called self-management support requiring a cooperative relationship between the patient, the family, and the professionals.

Explanation

CAT/2018.2(Verbal Ability)

Question. 47

The four sentences (labelled 1, 2, 3, 4) given below, when properly sequenced would yield a coherent paragraph. Decide on the proper sequence of the order of the sentences and key in the sequence of the four numbers as your answer.

1. They would rather do virtuous side projects assiduously as long as these would not compel them into doing their day jobs more honourably or reduce the profit margins.
2. They would fund a million of the buzzwordy programs rather than fundamentally question the rules of their game or alter their own behavior to reduce the harm of the existing distorted, inefficient and unfair rules.
3. Like the dieter who would rather do anything to lose weight than actually eat less, the business elite would save the world through social-impact-investing and philanthro-capitalism.
4. Doing the right thing — and moving away from their win-win mentality — would involve real sacrifice; instead, it’s easier to focus on their pet projects and initiatives. 

 

Explanation

CAT/2018.2(Verbal Ability)

Question. 48

The four sentences (labelled 1, 2, 3, 4) given below, when properly sequenced would yield a coherent paragraph. Decide on the proper sequence of the order of the sentences and key in the sequence of the four numbers as your answer.

1. In the era of smart world, however, ‘Universal Basic Income’ is an ineffective instrument which cannot address the potential breakdown of the social contract when large swathes of the population would effectively be unemployed.
2. In the era of industrial revolution, the abolition of child labour, poor laws and the growth of trade unions helped families cope with the pressures of mechanised work.
3. Growing inequality could be matched by a creeping authoritarianism that is bolstered by technology that is increasingly able to peer into the deepest vestiges of our lives.
4. New institutions emerge which recognise ways in which workers could contribute to and benefit by economic growth when, rather than if, their jobs are automated.

 

Explanation

CAT/2017.1(Verbal Ability)

Question. 49

The five sentences (labelled 1, 2, 3, 4 and 5) given below, when properly sequenced would yield a coherent paragraph. Decide on the proper sequence of the order of the sentences and key in the sequence of the five numbers as your answer.

1. The process of handing down implies not a passive transfer, but some contestation in defining what exactly is to be handed down.
2. Wherever Western scholars have worked on the Indian past, the selection is even more apparent and the inventing of a tradition much more recognizable.
3. Every generation selects what it requires from the past and makes its innovations, some more than others.
4. It is now a truism to say that traditions are not handed down unchanged, but are invented.
5. Just as life has death as its opposite, so is tradition by default the opposite of innovation.

Explanation

CAT/2017.1(Verbal Ability)

Question. 50

The five sentences (labelled 1, 2, 3, 4 and 5) given below, when properly sequenced would yield a coherent paragraph. Decide on the proper sequence of the order of the sentences and key in the sequence of the five numbers as your answer.

1. Scientists have for the first time managed to edit genes in a human embryo to repair a genetic mutation, fuelling hopes that such procedures may one day be available outside laboratory conditions.
2. The cardiac disease causes sudden death in otherwise healthy young athletes and affects about one in 500 people overall.
3. Correcting the mutation in the gene would not only ensure that the child is healthy but also prevents transmission of the mutation to future generations.
4. It is caused by a mutation in a particular gene and a child will suffer from the condition even if it inherits only one copy of the mutated gene.
5. In results announced in Nature this week, scientists fixed a mutation that thickens the heart muscle, a condition called hypertrophic cardiomyopathy.

 
 

Explanation

CAT/2017.1(Verbal Ability)

Question. 51

The five sentences (labelled 1, 2, 3, 4 and 5) given below, when properly sequenced would yield a coherent paragraph. Decide on the proper sequence of the order of the sentences and key in the sequence of the five numbers as your answer.

1. The study suggests that the disease did not spread with such intensity, but that it may have driven human migrations across Europe and Asia.
2. The oldest sample came from an individual who lived in southeast Russia about 5,000 years ago.
3. The ages of the skeletons correspond to a time of mass exodus from today's Russia and Ukraine into western Europe and central Asia, suggesting that a pandemic could have driven these migrations.
4. In the analysis of fragments of DNA from 101 Bronze Age skeletons for sequences from Yersinia pestis, the bacterium that causes the disease, seven tested positive.
5. DNA from Bronze Age human skeletons indicate that the black plague could have emerged as early as 3,000 BCE, long before the epidemic that swept through Europe in the mid-1300s.

Explanation

CAT/2017.1(Verbal Ability)

Question. 52

The five sentences (labelled 1, 2, 3, 4 and 5) given below, when properly sequenced would yield a coherent paragraph. Decide on the proper sequence of the order of the sentences and key in the sequence of the five numbers as your answer.

 

1. This visual turn in social media has merely accentuated this announcing instinct of ours, enabling us with easy-to-create, easy-to-share, easy-to-store and easy-to-consume platforms, gadgets and apps.
2. There is absolutely nothing new about us framing the vision of who we are or what we want, visually or otherwise, in our Facebook page, for example.
3. Turning the pages of most family albums, which belong to a period well before the digital dissemination of self-created and self-curated moments and images, would reconfirm the basic instinct of documenting our presence in a particular space, on a significant occasion, with others who matter.
4. We are empowered to book our faces and act as celebrities within the confinement of our respective friend lists, and communicate our activities, companionship and locations with minimal clicks and touches.
5. What is unprecedented is not the desire to put out news feeds related to the self, but the ease with which this broadcast operation can now be executed, often provoking (un)anticipated responses from beyond one's immediate location.

 
 

Explanation

CAT/2017.2(Verbal Ability)

Question. 53

The five sentences (labelled 1, 2, 3, 4 and 5) given below, when properly sequenced would yield a coherent paragraph. Decide on the proper sequence of the order of the sentences and key in the sequence of the five numbers as your answer.

1. Before plants can take life from atmosphere, nitrogen must undergo transformations similar to ones that food undergoes in our digestive machinery.
2. . In its aerial form nitrogen is insoluble, unusable and is in need of transformation.
3. Lightning starts the series of chemical reactions that need to happen to nitrogen, ultimately helping it nourish our earth.
4. Nitrogen — an essential food for plants — is an abundant resource, with about 22 million tons of it floating over each square mile of earth.
5. One of the most dramatic examples in nature of ill wind that blows goodness is lightning.

Explanation

CAT/2017.2(Verbal Ability)

Question. 54

The five sentences (labelled 1, 2, 3, 4 and 5) given below, when properly sequenced would yield a coherent paragraph. Decide on the proper sequence of the order of the sentences and key in the sequence of the five numbers as your answer.

1. This has huge implications for the health care system as it operates today, where depleted resources and time lead to patients rotating in and out of doctor's offices, oftentimes receiving minimal care or concern (what is commonly referred to as "bed side manner") from doctors.
2. The placebo effect is when an individual's medical condition or pain shows signs of improvement based on a fake intervention that has been presented to them as a real one and used to be regularly dismissed by researchers as a psychological effect.
3. The placebo effect is not solely based on believing in treatment, however, as the clinical setting in which treatments are administered is also paramount.
4. That the mind has the power to trigger biochemical changes because the individual believes that a given drug or intervention will be effective could empower chronic patients through the notion of our bodies' capacity for self-healing.
5. Placebo effects are now studied not just as foils for "real" interventions but as a potential portal into the self-healing powers of the body.

 
 

Explanation

CAT/2017.2(Verbal Ability)

Question. 55

The five sentences (labelled 1, 2, 3, 4 and 5) given below, when properly sequenced would yield a coherent paragraph. Decide on the proper sequence of the order of the sentences and key in the sequence of the five numbers as your answer.

1. Johnson treated English very practically, as a living language, with many different shades of meaning and adopted his definitions on the principle of English common law — according to precedent.
2. Masking a profound inner torment, Johnson found solace in compiling the words of a language that was, in its coarse complexity and comprehensive genius, the precise analogue of his character.
3. Samuel Johnson was a pioneer who raised common sense to heights of genius, and a man of robust popular instincts whose watchwords were clarity, precision and simplicity.
4. The 18th century English reader, in the new world of global trade and global warfare, needed a dictionary with authoritative acts of definition of words of a language that was becoming seeded throughout the first British empire by a vigorous and practical champion.
5. The Johnson who challenged Bishop Berkeley's solipsist theory of the nonexistence of matter by kicking a large stone ("I refute it thus") is the same Johnson for whom language must have a daily practical use.

Explanation

CAT/2017.2(Verbal Ability)

Question. 56

The five sentences (labelled 1, 2, 3, 4 and 5) given below, when properly sequenced would yield a coherent paragraph. Decide on the proper sequence of the order of the sentences and key in the sequence of the five numbers as your answer.

1. The implications of retelling of Indian stories, hence, takes on new meaning in a modern India.
2. The stories we tell reflect the world around us.
3. We cannot help but retell the stories that we value — after all, they are never quite right for us — in our time.
4. And even if we manage to get them quite right, they are only right for us — other people living around us will have different reasons for telling similar stories.
5. As soon as we capture a story, the world we were trying to capture has changed.

 
 

Explanation

CAT/2007(Verbal Ability)

Question. 57

In each question, there are five sentences/paragraphs. The sentence/paragraph labeled A is in its correct place. The four that follow are labeled B, C, D, and E, and need to be arranged in the logical order to form a coherent paragraph/ passage. From the given options, choose the most appropriate option.

A. In America, highly educated women, who are in a stronger positions in the labour market than less qualified ones, have higher rates of marriage than other groups.

B. Some work supports the Becker thesis, and some appear to contradict it.

C. And, as with crime, it is equally inconclusive.

D. But regardless of the conclusion of any particular piece of work, it is hard to establish convincing connections between family changes and economic factors using conventional approaches.

E. Indeed, just as with crime, an enormous academic literature exists on the validity of the pure economic approach to the evolution of family structures.

 

CAT/2007(Verbal Ability)

Question. 58

In each question, there are five sentences/paragraphs. The sentence/paragraph labeled A is in its correct place. The four that follow are labeled B, C, D, and E, and need to be arranged in the logical order to form a coherent paragraph/ passage. From the given options, choose the most appropriate option.

A. Personal experience of mothering and motherhood are largely framed in relation to two discernible or “official” discourses: the “medical discourse and natural childbirth discourse”. Both of these tend to focus on the “optimistic stories” of birth and mothering and underpin stereotypes of the “good mother”.

B. At the same time, the need for medical expert guidance is also a feature for contemporary reproduction and motherhood. But constructions of good mothering have not always been so conceived - and in different contexts may exist in parallel to other equally dominant discourses.

C. Similarly, historical work has shown how what are now taken-for-granted aspects of reproduction and mothering practices result from contemporary “pseudoscientific directives” and “managed constructs”. These changes have led to a reframing of modern discourses that pattern pregnancy and motherhood leading to an acceptance of the need for greater expert management.

D. The contrasting, overlapping, and ambiguous strands within these frameworks focus to varying degrees on a woman’s biological tie to her child and predisposition to instinctively know and be able to care for her child.

E. In addition, a third, “unofficial popular discourse” comprising “old wives” tales and based on maternal experiences of childbirth has also been noted. These discourses have also been acknowledged in work exploring the experiences of those who apparently do not “conform” to conventional stereotypes of the “good mother”.

CAT/2007(Verbal Ability)

Question. 59

In each question, there are five sentences/paragraphs. The sentence/paragraph labeled A is in its correct place. The four that follow are labeled B, C, D, and E, and need to be arranged in the logical order to form a coherent paragraph/ passage. From the given options, choose the most appropriate option.

A. Indonesia has experienced dramatic shifts in its formal governance arrangements since the fall of President Soeharto and the close of his centralized, authoritarian “New Order” regime in 1997.

B. The political system has taken its place in the nearly 10 years since Reformasi began. It has featured the active contest for political office among a proliferation of parties at central, provincial, and district levels; direct elections for the presidency (since 2004); and radical changes in center-local government relations towards administrative, fiscal, and political decentralization.

C. The mass media, once tidily under Soeharto’s thumb, has experienced significant liberalization, as has the legal basis for non-governmental organizations, including many dedicated to such controversial issues as corruption control and human rights.

D. Such developments are seen optimistically by a number of donors and some external analysts, who interpret them as signs of Indonesia’s political normalization.

E. A different group of analysts paints a picture in which the institutional forms have changed, but power relations have not. Vedi Hadiz argues that Indonesia’s “democratic transition” has been anything but linear.

CAT/2007(Verbal Ability)

Question. 60

In each question, there are five sentences/paragraphs. The sentence/paragraph labeled A is in its correct place. The four that follow are labeled B, C, D, and E, and need to be arranged in the logical order to form a coherent paragraph/ passage. From the given options, choose the most appropriate option.

A. I had six thousand acres of land, and had thus got much spare land besides the coffee plantation. Part of the farm was native forest, and about one thousand acres were squatters’ land, what (the Kikuyu) called their shambas.

B. The squatters’ land was more intensely alive than the rest of the farm, and was changing with the seasons the year round. The maize grew up higher than your head as you walked on the narrow hard-trampled footpaths in between the tall green rustling regiments.

C. The squatters are Natives, who with their families hold a few acres on a white man’s farm, and in return have to work for him a certain number of days in the year. My squatters, I think, saw the relationship in a different light, for many of them were born on the farm, and their fathers before them, and they very likely regarded me as a sort of superior squatter on their estates.

D. The Kikuyu also grew the sweet potatoes that have a vine like leaf and spread over the ground like a dense entangled mat, and many varieties of big yellow and green speckled pumpkins.

E. The beans ripened in the fields, were gathered and thrashed by the women, and the maize stalks and coffee pods were collected and burned, so that in certain seasons thin blue columns of smoke rose here and there all over the farm.

CAT/2005(Verbal Ability)

Question. 61

In the following question, the answer choices suggest alternative arrangements of four or more sentences (denoted by A, B, C, D, E, F). Choose the alternative which suggests a coherent paragraph.

A. Similarly, turning to caste, even though being lower caste is undoubtedly a separate cause of the disparity, its impact is all the greater when the lower-caste families also happen to be poor.

B. Belonging to a privileged class can help a woman to overcome many barriers that obstruct women from less thriving classes.

C. It is the interactive presence of these two kinds of deprivation - being low class and being female - that massively impoverishes women from the less privileged classes.

D. A congruence of class deprivation and gender discrimination can blight the lives of poorer women very severely.

E. Gender is certainly a contributor to societal inequality, but it does not act independently of class.

 

CAT/2005(Verbal Ability)

Question. 62

In the following question, the answer choices suggest alternative arrangements of four or more sentences (denoted by A, B, C, D, E, F). Choose the alternative which suggests a coherent paragraph.

A. This is now orthodoxy to which I subscribe - up to a point.

B. It emerged from the mathematics of chance and statistics.

C. Therefore, the risk is measurable and manageable.

D. The fundamental concept: Prices are not predictable, but the mathematical laws of chance can describe their fluctuations.

E. This is how what business schools now call modern finance was born.

CAT/2005(Verbal Ability)

Question. 63

In the following question, the answer choices suggest alternative arrangements of four or more sentences (denoted by A, B, C, D, E, F). Choose the alternative which suggests a coherent paragraph.

A. When identity is thus ‘defined by contrast’, divergence with the West becomes central.

B. Indian religious literature such as the Bhagavad Gita or the Tantric texts, which are identified as differing from secular writings seen as ‘western’, elicits much greater interest in the West than do other Indian writings, including India’s long history of heterodoxy.

C. There is a similar neglect of Indian writing on non-religious subjects, from mathematics, epistemology, and natural science to economics and linguistics.

D. Through selective emphasis that points up differences with the West, other civilizations can, in this way, be redefined in alien terms, which can be exotic and charming, or else bizarre and terrifying, or simply strange and engaging.

E. The exception is the Kamasutra in which western readers have managed to cultivate an interest.

CAT/2004(Verbal Ability)

Question. 64

In the following question, the answer choices suggest alternative arrangements of four or more sentences (denoted by A, B, C, D, E, F). Choose the alternative which suggests a coherent paragraph.

A. The two neighbours never fought each other.

B. Fights involving three male fiddler crabs have been recorded, but the status of the participants was unknown.

C. They pushed or grappled only with the intruder.

D. We recorded 17 cases in which a resident that was fighting an intruder was joined by an immediate neighbour, an ally.

E. We, therefore, tracked 268 intruder males until we saw them fighting a resident male.

CAT/2004(Verbal Ability)

Question. 65

In the following question, the answer choices suggest alternative arrangements of four or more sentences (denoted by A, B, C, D, E, F). Choose the alternative which suggests a coherent paragraph.

A. He felt justified in bypassing Congress altogether on a variety of moves.

B. At times he was fighting the entire Congress.

C. Bush felt he had a mission to restore power to the presidency.

D. Bush was not fighting just the democrats.

E. Representative democracy is a messy business, and a CEO of the White House does not like a legislature of second guessers and time wasters.

CAT/2004(Verbal Ability)

Question. 66

In the following question, the answer choices suggest alternative arrangements of four or more sentences (denoted by A, B, C, D, E, F). Choose the alternative which suggests a coherent paragraph.

A. In the west, Allied Forces had fought their way through southern Italy as far as Rome.

B. In June 1944 Germany’s military position in World War Two appeared hopeless.

C. In Britain, the task of amassing the men and materials for the liberation of northern Europe had been completed.

D. The Red Army was poised to drive the Nazis back through Poland.

E. The situation on the eastern front was catastrophic.

CAT/2004(Verbal Ability)

Question. 67

In the following question, the answer choices suggest alternative arrangements of four or more sentences (denoted by A, B, C, D, E, F). Choose the alternative which suggests a coherent paragraph. (2004 - 2 marks)

A. Expert such as Larry Burns, head of research at GM, reckons that only such a full-hearted leap will allow the world to cope with the mass motorization that will one day come to China or India. 

B. But once hydrogen is being produced from biomass or extracted from underground coal of made from water, using nuclear or renewable electricity, the way will be open for a huge reduction in carbon emissions from, the whole system.

C. In theory, once all the bugs have been sorted out, fuel cells should deliver better total fuel economy than any existing engines.

D. That is twice as good as the internal combustion engine, but only five percentage points better than a diesel hybrid.

E. Allowing for the resources needed to extract hydrogen from hydrocarbon, oil, coal or gas, the fuel cell has an efficiency of 30%.

CAT/2004(Verbal Ability)

Question. 68

In the following question, the answer choices suggest alternative arrangements of four or more sentences (denoted by A, B, C, D, E, F). Choose the alternative which suggests a coherent paragraph. (2004 - 2 marks)

A. But this does not mean that death was the Egyptians’ only preoccupation.

B. Even papyri come mainly from pyramid temples.

C. Most of our traditional sources of information about the Old Kingdom are monuments of the rich like pyramids and tombs.

D. House in which ordinary Egyptians lived have not been preserved, and when most people died they were buried in simple graves.

E. We know infinitely more about the wealthy people of Egypt than we do about the ordinary people, almost monuments were made for the rich.

CAT/2003(Verbal Ability)

Question. 69

In the following question, the answer choices suggest alternative arrangements of four or more sentences (denoted by A, B, C, D, E, F). Choose the alternative which suggests a coherent paragraph.

A. To avoid this, the QWERTY layout put the keys most likely to be hit in rapid succession on opposite sides. This made the keyboard slow, the story goes, but that was the idea.

B. A different layout, which had been patented by August Dvorak in 1936, was shown to be much faster.

C. The QWERTY design (patented by Christopher Sholes in 1868 and sold to Remington in 1873) aimed to solve a mechanical problem of early typewriters.

D. Yet the Dvorak layout has never been widely adopted, even though (with electric typewriters and then PCs) the antijamming rationale for QWERTY has been defunct for years.

E. When certain combinations of keys were struck quickly, the type bars often jammed.

CAT/2003(Verbal Ability)

Question. 70

In the following question, the answer choices suggest alternative arrangements of four or more sentences (denoted by A, B, C, D, E, F). Choose the alternative which suggests a coherent paragraph.

A. Surrendered, or captured, combatants cannot be incarcerated in razor wire cages; this ‘war’ has a dubious legality.

B. How can then one characterize a conflict to be waged against a phenomenon as war?

C. The phrase ‘war against terror’, which has passed into the common lexicon, is a huge misnomer.

D. Besides, war has a juridical meaning in international law, which has codified the laws of war, imbuing them with a humanitarian content.

E. Terror is a phenomenon, not an entity – either State or non-State.

CAT/2003(Verbal Ability)

Question. 71

In the following question, the answer choices suggest alternative arrangements of four or more sentences (denoted by A, B, C, D, E, F). Choose the alternative which suggests a coherent paragraph.

A. I am much more intolerant of a human being’s shortcomings than I am of an animal’s, but in this respect, I have been lucky, for most of the people I have come across have been charming.

B. Then you come across the unpleasant human animal – the District Officer who drawled, ‘We chaps are here to help you chaps’, and then proceeded to be as obstructive as possible.

C. In these cases, of course, the fact that you are an animal collector helps; people always seem delighted to meet someone with such an unusual occupation and go out of their way to assist you.

D. Fortunately, these types are rare, and the pleasant ones I have met more than compensated for them – but even so, I think I will stick to animals.

E. When you travel around the world collecting animals you also, of necessity, collect human beings.

CAT/2003(Verbal Ability)

Question. 72

In the following question, the answer choices suggest alternative arrangements of four or more sentences (denoted by A, B, C, D, E, F). Choose the alternative which suggests a coherent paragraph.

A. A few months ago I went to Princeton University to see what the young people who are going to be running our country in a few decades are like.

B. I would go to sleep in my hotel room around midnight each night, and when I awoke, my mailbox would be full of replies – sent at 1 : 15 a.m., 2 : 59 a.m., 3 : 23 a.m.

C. One senior told me that she went to bed around two and woke up each morning at seven; she could afford that much rest because she had learnt to supplement her full day of work by studying in her sleep.

D. Faculty members gave me the names of a few dozen articulate students, and I sent them e-mails, inviting them out to lunch or dinner in small groups.

E. As she was falling asleep she would recite a math problem or a paper topic to herself; she would then sometimes dream about it, and when she woke up, the problem might be solved.

CAT/2003(Verbal Ability)

Question. 73

In the following question, the answer choices suggest alternative arrangements of four or more sentences (denoted by A, B, C, D, E, F). Choose the alternative which suggests a coherent paragraph.

A. Four days later, Oracle announced its own bid for People Soft and invited the firm’s board to a discussion.

B. Furious that his own plans had been endangered. Peoplesoft’s boss, Craig Conway, called Oracle’s offer “diabolical”, and its boss, Larry Ellison, a “sociopath”.

C. In early June, People Soft said that it would buy J.D. Edwards, a smaller rival.

D. Moreover said Mr. Conway, he “could imagine no price nor combination of price and other condition to recommend accepting the offer”.

E. On June 12th, PeopleSoft turned Oracle down.

CAT/2003(Verbal Ability)

Question. 74

In the following question, the answer choices suggest alternative arrangements of four or more sentences (denoted by A, B, C, D, E, F). Choose the alternative which suggests a coherent paragraph.

A. The wall does not simply divide Israel from a putative Palestinian state on the basis of the 1967 borders.

B. A chilling omission from the road map is the gigantic ‘separation wall’ now being built in the West Bank by Israel.

C. It is surrounded by trenches, electric wire and moats; there are watchtowers at regular intervals.

D. It actually takes in new tracts of Palestinian land, sometimes five or six kilometers at a stretch.

E. Almost a decade after the end of South African apartheid, this ghastly racist wall is going up with scarcely a peep from Israel’s American allies who are going to pay for most of it.

CAT/2003(Verbal Ability)

Question. 75

In the following question, the answer choices suggest alternative arrangements of four or more sentences (denoted by A, B, C, D, E, F). Choose the alternative which suggests a coherent paragraph.

A. Luckily the tide of battle moved elsewhere after the American victory at Midway and an Australian victory over Japan at Milne Bay.

B. It could have been no more than delaying tactic.

C. The Australian Military, knowing the position was hopeless, planned to fall back to the south-east in the hope of defending the main cities.

D. They had captured most of the Solomon Islands and much of New Guinea, and seemed poised for an invasion.

E. Not many people outside Australia realize how close the Japanese got.

CAT/2003(Verbal Ability)

Question. 76

In the following question, the answer choices suggest alternative arrangements of four or more sentences (denoted by A, B, C, D, E, F). Choose the alternative which suggests a coherent paragraph.

A. Call it the third wave sweeping the Indian media.

B. Now, they are starring in a new role, as suave dealmakers who are in a hurry to strike alliances and agreements.

C. Look around and you will find a host of deals that have been inked or ready to be finalized.

D. Then the media barons wrested back control from their editors, and turned marketing warriors with the brand as their missile.

E. The first came with those magnificent men in their mahogany chambers who took on the world with their mighty fountain pens.

CAT/2003(Verbal Ability)

Question. 77

In the following question, the answer choices suggest alternative arrangements of four or more sentences (denoted by A, B, C, D, E, F). Choose the alternative which suggests a coherent paragraph.

A. The celebrations of economic recovery in Washington may be as premature as that “Mission Accomplished” banner hung on the USS Abraham Lincoln to hail the end of the Iraq war.

B. Meanwhile, in the real world, the struggles of families and communities continue unabated.

C. Washington responded to the favorable turn in economic news with enthusiasm.

D. The celebrations and high-fives up and down Pennsylvania Avenue are not to be found beyond the Beltway.

E. When the third quarter GDP showed growth of 7.2 % and the monthly unemployment rate dipped to 6%, euphoria gripped the US capital.

CAT/2003(Verbal Ability)

Question. 78

In the following question, the answer choices suggest alternative arrangements of four or more sentences (denoted by A, B, C, D, E, F). Choose the alternative which suggests a coherent paragraph.

A. To much of the labour movement, it symbolises the brutality of the upper classes.

B. And to everybody watching, the current mess over foxhunting symbolises the government’s weakness.

C. To foxhunting’s supporters, Labour’s 1991 manifesto commitment to ban it symbolises the party’s metropolitan roots and hostility to the countryside.

D. Small issues sometimes have large symbolic power.

E. To those who enjoy thundering across the countryside in redcoats after foxes, foxhunting symbolises the ancient roots of rural lives.

CAT/2003(Verbal Ability)

Question. 79

In the following question, the answer choices suggest alternative arrangements of four or more sentences (denoted by A, B, C, D, E, F). Choose the alternative which suggests a coherent paragraph.

A. In the case of king Merolchazzar’s courtship of the princess of the outer Isles, there occurs a regrettable hitch. 

B. She acknowledges the gifts, but no word of a meeting date follows.

C. The monarch hearing good reports of a neighbouring princess, dispatches messengers with gifts to her court, beseeching an interview.

D. The princess names a date, and a formal meeting takes place; after that everything buzzes along pretty smoothly.

E. Royal love affairs in olden days were conducted on the correspondence method.

CAT/2003(Verbal Ability)

Question. 80

In the following question, the answer choices suggest alternative arrangements of four or more sentences (denoted by A, B, C, D, E, F). Choose the alternative which suggests a coherent paragraph.

A. Who can trace to its first beginnings the love of Damon for Pythias, of David for Jonathan, of Swan for Edgar?

B. Similarly with men.

C. There is about great friendship between man and man a certain inevitability that can only be compared with the age old association of ham and eggs.

D. One simply feels that it is one of the things that must be so.

E. No one can say what was the mutual magnetism that brought the deathless partnership of these wholesome and palatable foodstuffs about.

CAT/2003(Verbal Ability)

Question. 81

In the following question, the answer choices suggest alternative arrangements of four or more sentences (denoted by A, B, C, D, E, F). Choose the alternative which suggests a coherent paragraph.

A. Events intervened, and in the late 1930s and 1940s. Germany suffered from “over-branding”.

B. The British used to be fascinated by the home of Romanticism.

C. But reunification and the federal government’s move to Berlin have prompted Germany to think again about its image.

D. The first foreign package holiday was a tour of Germany organized by Thomas Cook in 1855.

E. Since then, Germany has been understandably nervous about promoting itself abroad.

CAT/2002(Verbal Ability)

Question. 82

In the following question, the answer choices suggest alternative arrangements of four or more sentences (denoted by A, B, C, D, E, F). Choose the alternative which suggests a coherent paragraph.

A. Branded disposable diapers are available at many supermarkets and drug stores.

B. If one supermarket sets a higher price for a diaper, customers may buy that brand elsewhere.

C. By contrast, the demand for private-label products may be less price-sensitive since it is available only at a corresponding supermarket chain.

D. So, the demand for branded diapers at any particular store may be quite price sensitive.

E. For instance, only Save On Drugs stores sell Save On Drugs diapers.

F. Then, stores should set a higher incremental margin percentage for private-label diapers.

CAT/2002(Verbal Ability)

Question. 83

In the following question, the answer choices suggest alternative arrangements of four or more sentences (denoted by A, B, C, D, E, F). Choose the alternative which suggests a coherent paragraph.

A. Having a strategy is a matter of discipline.

B. It involves the configuration of a tailored value chain that enables a company to offer unique value.

C. It requires a strong focus on profitability and a willingness to make tough tradeoffs in choosing what not to do.

D. Strategy goes far beyond the pursuit of best practices.

E. A company must stay the course even during times of upheaval, while constantly improving and extending its distinctive positioning.

F. When a company’s activities fit together as a self-reinforcing system, any competitor wishing to imitate a strategy must replicate the whole system.

CAT/2002(Verbal Ability)

Question. 84

In the following question, the answer choices suggest alternative arrangements of four or more sentences (denoted by A, B, C, D, E, F). Choose the alternative which suggests a coherent paragraph.

A. As officials, their vision of a country shouldn’t run too far beyond that of the local people with whom they have to deal.

B. Ambassadors have to choose their words.

C. To say what they feel they have to say, they appear to be denying or ignoring part of what they know.

D. So, with ambassadors as with other expatriates in black Africa, there appears at a first meeting a kind of ambivalence.

E. They do a specialized job and it is necessary for them to live ceremonial lives.

CAT/2002(Verbal Ability)

Question. 85

In the following question, the answer choices suggest alternative arrangements of four or more sentences (denoted by A, B, C, D, E, F). Choose the alternative which suggests a coherent paragraph.

A. “This face off will continue for several months given the strong convictions on either side,” says a senior functionary of the high-powered task force on drought.

B. During the past week-and-half, the Central Government has sought to deny some of the earlier apprehensions over the impact of drought.

C. The recent revival of the rains had led to the emergence of a line of divide between the two.

D. The state governments, on the other hand, allege that the Centre is downplaying the crisis only to evade its full responsibility of financial assistance that is required to alleviate the damage.

E. Shrill alarm about the economic impact of an inadequate monsoon had been sounded by the Centre as well as most of the states, in late July and early August.

CAT/2002(Verbal Ability)

Question. 86

In the following question, the answer choices suggest alternative arrangements of four or more sentences (denoted by A, B, C, D, E, F). Choose the alternative which suggests a coherent paragraph.

A. This fact was established in the 1730s by French survey expeditions to Equador near the Equator and Lapland in the Arctic, which found that around the middle of the earth the arc was about a kilometer shorter.

B. One of the unsettled scientific questions in the late 18th century was the exact nature of the shape of the earth.

C. The length of a one-degree arc would be less near the equatorial latitudes than at the poles.

D. One way of doing that is to determine the length of the arc along a chosen longitude or meridian at one-degree latitude separation.

E. While it was generally known that the earth was not a sphere but an ‘oblate spheroid’, more curved at the equator and flatter at the poles, the question of ‘how much more’ was yet to be established.

CAT/2001(Verbal Ability)

Question. 87

In the following question, the answer choices suggest alternative arrangements of four or more sentences (denoted by A, B, C, D, E, F). Choose the alternative which suggests a coherent paragraph.

A. Although there are large regional variations, it is not infrequent to find a large number of people sitting here and there and doing nothing.

B. Once in office, they receive friends and relatives who feel free to call any time without a prior appointment.

C. While working, one is struck by the slow and clumsy actions and reactions, indifferent attitudes, procedure rather than outcome orientation, and the lack of consideration for others. D. Even those who are employed often come late to the office and leave early unless they are forced to be punctual.

E. Work is not intrinsically valued in India.

F. Quite often people visit ailing friends and relatives or go out of their way to help them in their personal matters even during office hours.

CAT/2001(Verbal Ability)

Question. 88

In the following question, the answer choices suggest alternative arrangements of four or more sentences (denoted by A, B, C, D, E, F). Choose the alternative which suggests a coherent paragraph.

A. But in the industrial era destroying the enemy’s productive capacity means bombing the factories which are located in the cities.

B. So in the agrarian era, if you need to destroy the enemy’s productive capacity, what you want to do is burn his fields, or if you’re really vicious, salt them.

C. Now in the information era, destroying the enemy’s productive capacity means destroying the information infrastructure.

D. How do you do battle with your enemy?

E. The idea is to destroy the enemy’s productive capacity, and depending upon the economic foundation, that productive capacity is different in each case.

F. With regard to defence, the purpose of the military is to defend the nation and be prepared to do battle with its enemy.

CAT/2001(Verbal Ability)

Question. 89

In the following question, the answer choices suggest alternative arrangements of four or more sentences (denoted by A, B, C, D, E, F). Choose the alternative which suggests a coherent paragraph.

A. Michael Hofman, a poet, and translator accepts this sorry fact without approval or complaint.

B. But thanklessness and impossibility do not daunt him.

C. He acknowledges too - in fact, he returns to the point often - that best translators of poetry always fail at some level.

D. Hofman feels passionate about his work, and this is clear from his writings.

E. In terms of the gap between worth and rewards, translators come somewhere near nurses and street - cleaners.

CAT/2001(Verbal Ability)

Question. 90

In the following question, the answer choices suggest alternative arrangements of four or more sentences (denoted by A, B, C, D, E, F). Choose the alternative which suggests a coherent paragraph.

A. Passivity is not, of course, universal.

B. In areas where there are no lords or laws, or in frontier zones where all men go armed, the attitude of the peasantry may well be different.

C. So indeed it may be on the fringe of the unsubmissive.

D. However, for most of the soil-bound peasants the problem is not whether to be normally passive or active, but when to pass from one state to another.

E. This depends on an assessment of the political situation. 

CAT/2001(Verbal Ability)

Question. 91

In the following question, the answer choices suggest alternative arrangements of four or more sentences (denoted by A, B, C, D, E, F). Choose the alternative which suggests a coherent paragraph.

A. The situations in which violence occurs and the nature of that violence tends to be clearly defined at least in theory, as in the proverbial Irishman’s question: ‘Is this a private fight or can anyone join in?’

B. So the actual risk to outsiders, though no doubt higher than our societies, is calculable.

C. Probably the only uncontrolled applications of force are those of social superiors to social inferiors and even here there are probably some rules.

D. However binding the obligation to kill, members of feuding families engaged in mutual massacre will be genuinely appalled if by some mischance a bystander or outsider is killed.

CAT/2000(Verbal Ability)

Question. 92

In the following question, the answer choices suggest alternative arrangements of four or more sentences (denoted by A, B, C, D, E, F). Choose the alternative which suggests a coherent paragraph.

A. If caught in the act, they were punished, not for the crime, but for allowing themselves to be caught another lash of the whip.

B. The bellicose Spartans sacrificed all the finer things in life for military expertise.

C. Those fortunate enough to survive babyhood were taken away from their mothers at the age of seven to undergo rigorous military training.

D. This consisted mainly of beatings and deprivations of all kinds like going around barefoot in winter, and worse starvation so that they would be forced to steal food to survive.

E. Male children were examined at birth by the city council and those deemed too weak to become soldiers were left to die of exposure.

 

CAT/2000(Verbal Ability)

Question. 93

In the following question, the answer choices suggest alternative arrangements of four or more sentences (denoted by A, B, C, D, E, F). Choose the alternative which suggests a coherent paragraph.

A. This very insatiability of the photographing eye changes the terms of confinement in the cave, our world.

B. Humankind lingers unregenerately in Plato’s cave, still reveling, its age-old habit, in mere images of truth.

C. But being educated by photographs is not like being educated by older images drawn by hand; for one thing, there are a great many more images around, claiming our attention.

D. The inventory started in 1839 and since then just about everything has been photographed, or so it seems.

E. In teaching us a new visual code. Photographs alter and enlarge our notions of what is worth looking at and what we have a right to observe.

CAT/2000(Verbal Ability)

Question. 94

In the following question, the answer choices suggest alternative arrangements of four or more sentences (denoted by A, B, C, D, E, F). Choose the alternative which suggests a coherent paragraph.

A. To be culturally literate is to possess the basic information needed to thrive in the modern world.

B. Nor is it confined to one social class; quite the contrary.

C. It is by no means confined to “culture” narrowly understood as an acquaintance with the arts.

D. Cultural literacy constitutes the only sure avenue of opportunity for disadvantaged children, the only reliable way of combating the social determinism that now condemns them.

E. The breadth of that information is great, extending over the major domains of human activity from sports to science.

CAT/2000(Verbal Ability)

Question. 95

In the following question, the answer choices suggest alternative arrangements of four or more sentences (denoted by A, B, C, D, E, F). Choose the alternative which suggests a coherent paragraph.

A. Both parties use capital and labour in the struggle to secure property rights.

B. The thief spends time and money in his attempt to steal (he buys wire cutters) and the legitimate property owner expends resources to prevent the theft (he buys locks).

C. A social cost of theft is that both the thief and the potential victim use resources to gain or maintain control over the property.

D. These costs may escalate as a type of technological arms race unfolds.

CAT/2000(Verbal Ability)

Question. 96

In the following question, the answer choices suggest alternative arrangements of four or more sentences (denoted by A, B, C, D, E, F). Choose the alternative which suggests a coherent paragraph.

A. The likelihood of an accident is determined by how carefully the motorist drives and how carefully the pedestrian crosses the street.

B. An accident involving a motorist and a pedestrian is such a case.

C. Each must decide how much care to exercise without knowing how careful the other is.

D. The simplest strategic problem arises when two individuals interact with each other, and each must decide what to do without knowing what the other is doing.

CAT/2000(Verbal Ability)

Question. 97

DIRECTIONS FOR QUESTIONS: Arrange sentences A, B, C, and D between sentences 1 and 6 to form a logical sequence of the six sentences.

1. Security inks exploit the same principle that causes the vivid and constantly changing colours of a film of oil on water. 

A. When two rays of light meet each other after being reflected from these different surfaces, they have each traveled slightly different distances.

B. The key is that the light is bouncing off two surface, that of the oil and that of the water layer below it.

C. The distance the two rays determines which wavelengths, and hence colours, interfere constructively and look bright.

D. Because light is an electromagnetic wave, the peaks and troughs of each ray then interfere either constructively, to appear bright, or destructively, to appear dim.

6. Since the distance the rays travel changes with the angle as you look at the surface, different colours look bright from different viewing angles. 

CAT/2000(Verbal Ability)

Question. 98

DIRECTIONS FOR QUESTIONS: Arrange sentences A, B, C, and D between sentences 1 and 6 to form a logical sequence of the six sentences.

1. Commercially reared chicken can be unusually aggressive, and are often kept in darkened sheds to prevent them pecking at each other. 

A. The birds spent far more of their time - up to a third - pecking at the inanimate objects in the pens, in contrast to birds in other pens which spent a lot of time attacking others.

B. In low light conditions, they behave less belligerently, but are more prone to opthalmic disorders and respiratory problems.

C. In an experiment, aggressive head-pecking was all but eliminated among birds in the enriched environment.

D. Altering the birds’ environment, by adding bales of wood-shavings to their pens, can work wonders.

6. Bales could diminish aggressiveness and reduce injuries; they might even improve productivity, since a happy chicken is a productive chicken.

CAT/2000(Verbal Ability)

Question. 99

DIRECTIONS FOR QUESTIONS: Arrange sentences A, B, C, and D between sentences 1 and 6 to form a logical sequence of the six sentences.

1. The concept of a ‘nation-state’ assumes a complete correspondence between the boundaries of the nation and the boundaries of those who live in a specific state. 

A. Then there are members of national collectivities who live in other countries, making a mockery of the concept.

B. There are always people living in particular states who are not considered to be (and often do not consider themselves to be) members of the hegemonic nation.

C. Even worse, there are nations which never had a state or which are divided across several states.

D. This, of course has been subject to severe criticism and is virtually everywhere a fiction.

6. However, the fiction has been, and continues to be, at the basic nationalist ideologies. 

CAT/2000(Verbal Ability)

Question. 100

DIRECTIONS FOR QUESTIONS: Arrange sentences A, B, C, and D between sentences 1 and 6 to form a logical sequence of the six sentences.

1. In the sciences, even questionable examples of research fraud are harshly punished. 

A. But no such mechanism exists in the humanities-much of what humanities researchers call research does not lead to results that are replicable by other scholars.

B. Given the importance of interpretation in historical and literary scholarship, humanities researchers are in a position where they can explain away deliberate and even systematic distortion.

C. Mere suspicion is enough for funding to be cut off ; publicity guarantees that careers can be effectively ended.

D. Forgeries which take the form of pastiches in which the forger intersperses fake and real parts can be defended as mere mistakes or aberrant misreading.

6. Scientists funding data have no such defences. (a) (b) (c) (d) 

CAT/2000(Verbal Ability)

Question. 101

DIRECTIONS FOR QUESTIONS: Arrange sentences A, B, C, and D between sentences 1 and 6 to form a logical sequence of the six sentences.

1. Horses and communism were, on the whole, a poor match. (2000)

A. Fine horses bespoke the nobility the party was supposed to despise.

B. Communist leaders, when they visited villages, preferred to see cows and pigs.

C. Although a working horse was just about tolerable, the communists were right to be wary.

D. Peasants from Poland to the Hungarian Pustza preferred their horse to party dogma.

6. “A farmer’s pride is his horse; his cow may be thin but his horse must be fat,” went a Slovak saying. (a) (b) (c) (d) 

CAT/1999(Verbal Ability)

Question. 102

In the following question, the answer choices suggest alternative arrangements of four or more sentences (denoted by A, B, C, D, E, F). Choose the alternative which suggests a coherent paragraph.

A. In rejecting the functionalism in positivist organization theory, either wholly or partially, there is often a move towards a political model of organization theory.

B. Thus the analysis would shift to the power resources possessed by different groups in held to be completely unrelated to the work to be done and to be caused totally by the political pursuit of self-interest.

C. At the extreme, in one set of writings, the growth of administrators in the organization is held to be completely unrelated to the work to be done and to be caused totally by the political pursuit of self-interest.

D. The political model holds that individual interests are pursued in organizational life through the exercise of power and influence.

CAT/1999(Verbal Ability)

Question. 103

In the following question, the answer choices suggest alternative arrangements of four or more sentences (denoted by A, B, C, D, E, F). Choose the alternative which suggests a coherent paragraph.

A. Group decision-making, however, does not necessarily fully guard against arbitrariness and anarchy, for individual capriciousness can get substituted by collusion of group members.

B. Nature itself is an intricate system of checks and balances meant to preserve the delicate balance between various environmental factors that affect our ecology.

C. In institutions also, there is a need to have in place a system of checks and balances which inhibits the concentration of power in only some individuals.

D. When human interventions alter this delicate balance, the outcomes have been seen to be disastrous.

CAT/1999(Verbal Ability)

Question. 104

In the following question, the answer choices suggest alternative arrangements of four or more sentences (denoted by A, B, C, D, E, F). Choose the alternative which suggests a coherent paragraph.

A. He was bone-weary and soul-weary, and found himself muttering, “ Either I can’t manage this place, or “it’s unmanageable.”

B. To his horror, he realized that he had become the victim of an amorphous, unwitting, unconscious conspiracy to immerse him in routine work that had no significance.

C. It was one of those nights in the office when the office clock was moving towards four in the morning and Bennis was still not through with the incredible mass of paper stacked before him.

D. He reached for his calendar and ran his eyes down each hour, half-hour, and quarter-hour to see where his time had gone that day, the day before, the month before.

CAT/1999(Verbal Ability)

Question. 105

In the following question, the answer choices suggest alternative arrangements of four or more sentences (denoted by A, B, C, D, E, F). Choose the alternative which suggests a coherent paragraph.

A. With that, I swallowed the shampoo and obtained the most realistic results almost on the spot.

B. The man shuffled away into the back regions to make up a prescription, and after a moment I got through on the shop telephone to the Consulate, intimating my location.

C. Then, while the pharmacist was wrapping up a six-ounce bottle of the mixture, I groaned and inquired whether he could give me something for acute gastric cramp.

D. I intended to stage a sharp gastric attack, and entering an old-fashioned pharmacy I asked for a popular shampoo mixture, consisting of olive oil and flaked soap.

CAT/1999(Verbal Ability)

Question. 106

In the following question, the answer choices suggest alternative arrangements of four or more sentences (denoted by A, B, C, D, E, F). Choose the alternative which suggests a coherent paragraph.

A. Since then, intelligence tests have been mostly used to separate dull children in school from average or bright children, so that special education can be provided to the dull.

B. In other words, intelligence tests give us a norm for each age.

C. Intelligences expressed as intelligence quotient and tests are developed to indicate what an average child of a certain age can do--what a 5 year old can answer, but a 4-year-old cannot, for instance.

D. Binet developed the first set of such tests in the early 1900s to find out which children in school needed special attention.

E. Intelligence can be measured by tests.

CAT/1999(Verbal Ability)

Question. 107

DIRECTIONS FOR QUESTIONS: Arrange sentences A, B, C, and D between sentences 1 and 6 to form a logical sequence of the six sentences.

1. Making people laugh is tricky. 

A. At times, the intended humour may not simply not come off.

B. Making people laugh while trying to sell them something is a tougher challenge, since the commercial can fall flat on two grounds.

C. There are many advertisements which do amuse but do not even begin to set the cash thrills ringing.

D. Again, it is rarely sufficient for an advertiser simply to amuse the target audience in order to reap the sales benefit.

6. There are indications that in substituting the hard sell for more entertaining approach, some agencies have rather thrown out the baby with the bath water. 

CAT/1999(Verbal Ability)

Question. 108

DIRECTIONS FOR QUESTIONS: Arrange sentences A, B, C, and D between sentences 1 and 6 to form a logical sequence of the six sentences.

1. Picture a termite colony, occupying a tall mud hump on an African plain. 

A. Hungry predators often invade the colony and unsettle the balance.

B. The colony flourishes only if the portion of soldiers to workers remain roughly the same, so that the queen and the workers can be protected by the soldiers, and the queen and the soldiers can be serviced by workers.

C. But its fortunes are presently restored, because the immobile queen walled in well below ground level, lays eggs not only large enough numbers, but also in the varying proportions required.

D. The hump is alive with worker termites and solider termites going about their distinct kind of business.

6. How can we account for her mysterious ability to respond like this to events on the distant surface.  

CAT/1999(Verbal Ability)

Question. 109

DIRECTIONS FOR QUESTIONS: Arrange sentences A, B, C, and D between sentences 1 and 6 to form a logical sequence of the six sentences.

1. According to recent research, the critical period for developing language skills is between the ages of three and five and a half years. 

A. The read-to child already has a large vocabulary and a sense of grammar and sentence structure.

B. Children who read in these years have a far better chance of reading well in school, indeed, of doing well in all subjects.

C. And the reason is actually quiet simple.

D. This correlation is far and away the highest yet found between home influences and school success.

6. Her comprehension of language is therefore is very high.

CAT/1999(Verbal Ability)

Question. 110

DIRECTIONS FOR QUESTIONS: Arrange sentences A, B, C, and D between sentences 1 and 6 to form a logical sequence of the six sentences.

1. High-powered outboard motors were considered to be one of the major threats to the survival of Beluga whale.

A. With these. hunters could approach belugas within hunting range and profit from its inner skin and blubber.

B. To escape an approaching motor, Belugas have learned to drive to the ocean bottom and stay there for up to 20 minutes, by which time the confused predator has left.

C. Today however, even with much more powerful engines, it is difficult to come close because the whales seem to disappear suddenly just when you thought you had them in your sights.

D. When the first outboard engines arrived in the early 1930’s. one came across 4 and 8 HP motors.

6. Belugas seem to have used their well- known sensitivity to noise to evolve an ‘avoidance’ strategy to outsmart hunters and their powerful technologies. 

CAT/1999(Verbal Ability)

Question. 111

DIRECTIONS FOR QUESTIONS: Arrange sentences A, B, C, and D between sentences 1 and 6 to form a logical sequence of the six sentences.

1. The reconstruction of history by post- revolutionary science texts involve more than a multiplication of historical misconstructions. 

A. Because they aim quickly to acquaint the student with the contemporary scientific community thinks it knows textbooks treat the various experiments, concepts, laws and theories of the current normal science as separately and as nearly seriatim as possible.

B. Those misconstruction render revolutions invisible; the arrangement of the still visible material in science texts implies a process that, if it existed, would deny revolutions a functions.

C. But when combined with the generally unhistorical air of science writing and with the occasional systematic misconstruction, one impression is likely to follow.

D. As pedagogy this technique of presentation is unexceptionable.

6. Science has reached its present state by series of individual discoveries and inventions that, when gathered together, constitute the modern body of technical knowledge. 

CAT/1998(Verbal Ability)

Question. 112

In the following question, the answer choices suggest alternative arrangements of four or more sentences (denoted by A, B, C, D, E, F). Choose the alternative which suggests a coherent paragraph.

A. He was carrying his jacket and walked with his head thrown back.

B. As Anette neared the lamp she saw a figure walking slowly.

C. For a while Michael walked on and she followed twenty paces behind.

D. With a mixture of terror and triumph of recognition she slackened her pace.

CAT/1998(Verbal Ability)

Question. 113

In the following question, the answer choices suggest alternative arrangements of four or more sentences (denoted by A, B, C, D, E, F). Choose the alternative which suggests a coherent paragraph.

A. However, the real challenge today is in unlearning, which is much harder.

B. But the new world of business behaves differently from the world in which we grew up.

C. Learning is important for both people and organizations.

D. Each of us has a ‘mental model’ that we’ve used over the years to make sense.

CAT/1998(Verbal Ability)

Question. 114

In the following question, the answer choices suggest alternative arrangements of four or more sentences (denoted by A, B, C, D, E, F). Choose the alternative which suggests a coherent paragraph.

A. There was nothing quite like a heavy downpour of rain to make life worthwhile.

B. We reached the field, soaked to the skin, and surrounded it.

C. The wet, as far as he was concerned, was ideal.

D. There, sure enough, stood Claudius, looking like a debauched Roman emperor under a shower.

CAT/1998(Verbal Ability)

Question. 115

In the following question, the answer choices suggest alternative arrangements of four or more sentences (denoted by A, B, C, D, E, F). Choose the alternative which suggests a coherent paragraph.

A. Alex had never been happy with his Indian origins.

B. He set about rectifying this grave injustice by making his house in his own image of a country manor.

C. Fate had been unfair to him; if he had his wish, he would have been a count or an Earl on some English estate, or a medieval monarch in a chateau in France.

D. This illusion of misplaced grandeur, his wife felt, would be Alex’s undoing.

CAT/1998(Verbal Ability)

Question. 116

In the following question, the answer choices suggest alternative arrangements of four or more sentences (denoted by A, B, C, D, E, F). Choose the alternative which suggests a coherent paragraph.

A. The influence is reflected the most in beaded wear.

B. Increasingly, the influence of India’s colors and cuts can be seen in western styles.

C. And even as Nehru jackets and jodhpurs remain staples of the fashion world, designers such as Armani and McFadden have turned to the sleek silhouette of the churidar this year.

D. Indian hot pink, paprika, and saffron continue to be popular colors year in and year out.

CAT/1998(Verbal Ability)

Question. 117

In the following question, the answer choices suggest alternative arrangements of four or more sentences (denoted by A, B, C, D, E, F). Choose the alternative which suggests a coherent paragraph.

A. Such a national policy will surely divide and never unite the people.

B. In fact, it suits the purpose of the politicians; they can drag the people into submission by appealing to them in the name of religion.

C. In order to inculcate the unquestioning belief they condemn the other states which do not follow their religion.

D. The emergence of the theocratic states where all types of crimes are committed in the name of religion has revived the religion of the Middle Ages.

CAT/1998(Verbal Ability)

Question. 118

In the following question, the answer choices suggest alternative arrangements of four or more sentences (denoted by A, B, C, D, E, F). Choose the alternative which suggests a coherent paragraph.

A. His left hand concealed a blackjack, his right hand groped for the torch in his pocket.

B. The meeting was scheduled for nine o’clock, and his watch showed the time to be a quarter to nine.

C. The man lurked in the corner, away from the glare of the light.

D. His heart thumped in his chest, sweat beads formed themselves on his forehead, his mouth was dry.

CAT/1998(Verbal Ability)

Question. 119

In the following question, the answer choices suggest alternative arrangements of four or more sentences (denoted by A, B, C, D, E, F). Choose the alternative which suggests a coherent paragraph.

A. The Director walked into the room and took a look around the class.

B. Mitch wanted to scream - the illogicality of the entire scene struck him dumb.

C. The managers stared at him with the look of fear that no democratic country should tolerate its people.

D. Mitch walked out of the room - it was his irrevocable protest against an insensible and insensitive situation.

CAT/1998(Verbal Ability)

Question. 120

In the following question, the answer choices suggest alternative arrangements of four or more sentences (denoted by A, B, C, D, E, F). Choose the alternative which suggests a coherent paragraph.

A. The establishment of the Third Reich influenced events in American history by starting a chain of events that culminated in war between Germany and the United States.

B. The Neutrality Acts of 1935 and 1936 prohibited trade with any belligerents or loans to them.

C. While speaking out against Hitler’s atrocities, the American people generally favoured isolationist policies and neutrality.

D. The complete destruction of democracy, the persecution of Jews, the war on religion, the cruelty and barbarism of the allies, caused great indignation in this country and brought on fear of another World War.

CAT/1998(Verbal Ability)

Question. 121

In the following question, the answer choices suggest alternative arrangements of four or more sentences (denoted by A, B, C, D, E, F). Choose the alternative which suggests a coherent paragraph.

A. An essay that appeals chiefly to the intellect is Francis Bacon’s of Studies.

B. His careful tripartite division of studies expressed succinctly in aphoristic prose demands the complete attention of the mind of the reader.

C. He considers studies as they should be; for pleasure, for self-improvement, for business.

D. He considers the evils of excess study; laziness, affectation, and preciosity.

CAT/1998(Verbal Ability)

Question. 122

In the following question, the answer choices suggest alternative arrangements of four or more sentences (denoted by A, B, C, D, E, F). Choose the alternative which suggests a coherent paragraph.

A. By reasoning we mean the mental process of drawing an inference from two or more statements or going from the inference to the statements which yield that inference.

B. So logical reasoning covers those types of questions that imply drawing an inference from the problems.

C. Logic means, if we take its original meaning, the science of valid reasoning

D. Clearly, for understanding arguments and for drawing the inference correctly it is necessary that we should understand the statements first.

CAT/1998(Verbal Ability)

Question. 123

DIRECTIONS FOR QUESTIONS: Arrange sentences A, B, C, and D between sentences 1 and 6 to form a logical sequence of the six sentences.

1. Buddhism is a way to salvation. 

A. But Buddhism is more severely analytical.

B. In the Christian tradition there is also a concern for the fate of human society conceived as a whole, rather than merely as a sum or network of individuals.

C. Salvation is a property, or achievement of individuals.

D. Not only does it dissolve society into individuals, the individual in turn is dissolved into component parts and instants, a stream of events.

6. In modern terminology, Buddhist doctrine is reductionist. 

CAT/1998(Verbal Ability)

Question. 124

DIRECTIONS FOR QUESTIONS: Arrange sentences A, B, C, and D between sentences 1 and 6 to form a logical sequence of the six sentences.

1. The problem of improving Indian agriculture is both a sociological and an administrative one. 

A. It also appears that there is a direct relationship between the size of a state and development.

B. The issues of Indian development, and the problems of India’s agricultural sector, will remain with us long into the next century.

C. Without improving Indian agriculture, no liberalisation and delicensing will be able to help India.

D. At the end of the day, there has to be a ferment and movement of life and action in the vast segment of rural India.

6. When it starts marching, India will fly. 

CAT/1998(Verbal Ability)

Question. 125

DIRECTIONS FOR QUESTIONS: Arrange sentences A, B, C, and D between sentences 1 and 6 to form a logical sequence of the six sentences.

1. Good literary magazines have always been good because of their editors. 

A. Furthermore, to edit by committee, as it were, would prevent any magazine from finding its own identity.

B. The more quirky and idiosyncratic they have been, the better the magazine is, at least as a general rule.

C. But the number of editors one can have for a magazine should also be determined by the number of contributions to it.

D. To have four editors for an issue that contains only seven contributions, is a bit silly to start with.

6. However, in spite of this anomaly, the magazine does acquire merit in its attempt to give a comprehensive view of the Indian literary scene as it is today.

CAT/1998(Verbal Ability)

Question. 126

DIRECTIONS FOR QUESTIONS: Arrange sentences A, B, C, and D between sentences 1 and 6 to form a logical sequence of the six sentences.

1. It’s the success story of the Indian expatriate in the US which today hogs much of the media coverage in India. 

A. East and West, the twain have met quite comfortably in their person, thank you.

B. Especially in its more recent romancing the -NRI phase.

C. Seldom does the price of getting there - more like not getting there - or what’s going on behind those sunny smiles get so much media hype.

D. Well groomed, with their perfect Colgate smiles, and hair in place, they appear the picture of confidence which comes from having arrived.

6. The festival of feature films and documentaries made by Americans of Indian descent being screened this fortnight, goes a long way in filling those gaps. 

CAT/1998(Verbal Ability)

Question. 127

DIRECTIONS FOR QUESTIONS: Arrange sentences A, B, C, and D between sentences 1 and 6 to form a logical sequence of the six sentences.

1. A market for Indian art has existed ever since the international art scene sprang to life. 

A. But interest in architectural conceits is an unanticipated fallout of the Festivals of India of the ‘80s’ which were designed to increase exports of Indian crafts.

B. Simultaneously, the Indian elite discarded their synthetic sarees and kitsch plastic furniture and a market came into being.

C. Western dealers, unhappy in a market afflicted by violent price fluctuations and unpredictable profit margins, began to look East, and found cheap antiques with irresistible appeal.

D. The fortunes of the Delhi supremos, the Jew Town dealers in Cochin and myriad others around the country were made.

6. A chain of command was established, from the local contacts to the provincial dealers and up to the big boys, who entertain the Italians and the French, cutting deals worth lakhs in warehouse worth corers. 

CAT/1997(Verbal Ability)

Question. 128

In the following question, the answer choices suggest alternative arrangements of four or more sentences (denoted by A, B, C, D, E, F). Choose the alternative which suggests a coherent paragraph.

A. It begins with an ordinary fever and a moderate cough.

B. India could be under attack from a class of germs that cause what is called atypical pneumonia.

C. Slowly, a sore throat progresses to bronchitis and then pneumonia and respiratory complications.

D. It appears like the ordinary flu, but baffled doctors find that the usual drugs don’t work.

CAT/1997(Verbal Ability)

Question. 129

In the following question, the answer choices suggest alternative arrangements of four or more sentences (denoted by A, B, C, D, E, F). Choose the alternative which suggests a coherent paragraph.

A. Chemists mostly don’t stock it: only a few government hospitals do but in limited quantities.

B. Delhi’s building boom is creating a bizarre problem: snakes are increasingly biting people as they emerge from their disturbed underground homes.

C. There isn’t enough anti-snake serum, largely because there is no centralized agency that distributes the product.

D. If things don’t improve, more people could face paralysis, and even death.

CAT/1997(Verbal Ability)

Question. 130

In the following question, the answer choices suggest alternative arrangements of four or more sentences (denoted by A, B, C, D, E, F). Choose the alternative which suggests a coherent paragraph.

A. But the last decade has witnessed greater voting and political participation by various privileged sections.

B. If one goes by the earlier record of mid-term elections, it is likely that the turnout in 1998 will drop by anything between four and six percentage points over the already low polling of 58 per cent in 1996.

C. If this trend offsets the mid-term poll fatigue, the fall may not be so steep.

D. Notwithstanding a good deal of speculation on this issue, it is still not clear that who benefits from a lower turnout.

CAT/1997(Verbal Ability)

Question. 131

In the following question, the answer choices suggest alternative arrangements of four or more sentences (denoted by A, B, C, D, E, F). Choose the alternative which suggests a coherent paragraph.

A. After several routine elections there comes to a ‘critical’ election that redefines the basic pattern of political loyalties, redraws political geography, and opens up the frozen political space.

B. In psephological jargon they call it realignment.

C. Rather, since 1989, there have been a series of semi-critical elections.

D. On a strict definition, none of the recent Indian elections qualifies as a critical election. 

CAT/1997(Verbal Ability)

Question. 132

In the following question, the answer choices suggest alternative arrangements of four or more sentences (denoted by A, B, C, D, E, F). Choose the alternative which suggests a coherent paragraph.

A. Trivial pursuits, marketed by the Congress, is a game imported from Italy.

B. The idea is to create an imaginary saviour in times of crisis so that the party doesn’t fall flat - on its collective face.

C. Closest contenders are Mani Shankar Aiyar, who still hears His Master’s Voice, and V. George, who is frustrated by the fact that his political future remains Sonia and yet so far.

D. The current champion is Arjun Singh for whom all roads lead to Rome, or in his case, 10 Janpath.

CAT/1997(Verbal Ability)

Question. 133

In the following question, the answer choices suggest alternative arrangements of four or more sentences (denoted by A, B, C, D, E, F). Choose the alternative which suggests a coherent paragraph.

A. Good advertising can make people buy your products even if it sucks.

B. A dollar spent on brainwashing is more cost-effective than a dollar spent on product improvement.

C. That’s important because it takes the pressure off you to make good products.

D. Obviously, there’s a minimum quality that every product has to achieve: it should be able to withstand the shipping process without becoming unrecognizable.

CAT/1997(Verbal Ability)

Question. 134

In the following question, the answer choices suggest alternative arrangements of four or more sentences (denoted by A, B, C, D, E, F). Choose the alternative which suggests a coherent paragraph.

A. Almost a century ago, when the father of the modern automobile industry, Henry Ford, sold the first Model A car, he decided that only the best would do for his customers.

B. Today, it is committed to delivering the finest quality with over six million vehicles a year in over 200 countries across the world.

C. And for over ninety years, this philosophy has endured in the Ford Motor Company.

D. A vehicle is ready for the customer only if it passes the Ford ‘Zero Defect Programme’.

CAT/1997(Verbal Ability)

Question. 135

In the following question, the answer choices suggest alternative arrangements of four or more sentences (denoted by A, B, C, D, E, F). Choose the alternative which suggests a coherent paragraph.

A. But, clearly, the government still has the final say.

B. In the past few years, the Reserve Bank of India might have wrested considerable powers from the government when it comes to monetary policy.

C. The RBI’s announcements on certain issues become effective only after the government notifies them.

D. Isn’t it time the government vested the RBI with powers to sanction such changes, leaving their ratification for later?

CAT/1997(Verbal Ability)

Question. 136

In the following question, the answer choices suggest alternative arrangements of four or more sentences (denoted by A, B, C, D, E, F). Choose the alternative which suggests a coherent paragraph.

A. I sat there frowning at the checkered table cloth, chewing the bitter cud of insight.

B. That wintry afternoon in Manhattan, waiting in the little French restaurant, I was feeling frustrated and depressed.

C. Even the prospect of seeing a dear friend failed to cheer me as it usually did.

D. Because of certain miscalculations on my part, a project of considerable importance in my life had fallen through.

CAT/1997(Verbal Ability)

Question. 137

In the following question, the answer choices suggest alternative arrangements of four or more sentences (denoted by A, B, C, D, E, F). Choose the alternative which suggests a coherent paragraph.

A. Perhaps the best known is the Bay Area Writing Project, founded by James Gray in 1974.

B. The decline in writing skills can be stopped.

C. Today’s back-to-basics movement has already forced some schools to place renewed emphasis on the three Rs.

D. Although the inability of some teachers to teach writing successfully remains a big stumbling block, a number of programmes have been developed to attack this problem.

CAT/1997(Verbal Ability)

Question. 138

DIRECTIONS FOR QUESTIONS: Arrange sentences A, B, C, and D between sentences 1 and 6 to form a logical sequence of the six sentences.

1. Whenever technology has flowered, it has put man’s language-developing skills into overdrive 

A. Technical and technoid terms are spilling into the mainstream almost as fast as junk-mail is slapped into e-mail boxes.

B. The era of computers is no less.

C. From the wheel with its axle to the spinning wheel with its bobbins, to the compact disc and its jewel-box, inventions have trailed new words in their wake.

D. “Cyberslang is huge, but it’s parochial, and we don’t know what will filter into the larger culture” said Tom Dalzell, who wrote the slang dictionary Flappers 2 Rappers.

6. Some slangs already have a pedigree. (a) (b) (c) (d) 

CAT/1997(Verbal Ability)

Question. 139

DIRECTIONS FOR QUESTIONS: Arrange sentences A, B, C, and D between sentences 1 and 6 to form a logical sequence of the six sentences.

1. Until the MBA arrived on the scene the IIT graduate was king (1997)

A. A degree from one of the five IIT’s was a passport to a well-paying job, great prospects abroad and, for some, a decent dowry to boot.

B. From the day he or she cracked the joint entrance exam, the IIT student commanded the awe of neighbours and close relatives.

C. IIT students had, meanwhile, also developed their own special culture, complete with lingo and attitude, which they passed down.

D. True, the success stories of IIT graduates are legion and they now constitute the cream of the Indian diaspora.

6. But not many alumni would agree that the IIT undergraduate mindset merits a serious psychological study, let alone an interactive one. (a) (b) (c) (d) 

CAT/1997(Verbal Ability)

Question. 140

DIRECTIONS FOR QUESTIONS: Arrange sentences A, B, C, and D between sentences 1 and 6 to form a logical sequence of the six sentences.

1. Some of the maharajas, like the one at Kapurthala, had exquisite taste 

A. In 1902, the Maharaja of Kapurthala gave his civil engineer photographs of the Versailles palace and asked him to replicate it, right down to the gargoyles.

B. Yeshwantrao Holkar of Indore brought in Bauhaus aesthetics and even works of modern artists like Brancusi and Duchamp.

C. Kitsch is the most polite way to describe them.

D. But many of them, as the available-light photographs show, had execrable taste.

6. Like Ali Baba’s caves, some of the palaces were like warehouses with the downright ugly next to the sublimely aesthetic.  

CAT/1997(Verbal Ability)

Question. 141

DIRECTIONS FOR QUESTIONS: Arrange sentences A, B, C, and D between sentences 1 and 6 to form a logical sequence of the six sentences.

1. There, in Europe, his true gifts unveiled 

A. Playing with Don Cherie, blending Indian music and jazz for the first time, he began setting the pace in the late 70s for much of present-day fusion is.

B. John McLaughlin, the legendary guitarist whose soul has always had an Indian stamp on it, was seduced immediately.

C. Fusion by Gurtu had begun.

D. He partnered Gurtu for four years, and ‘nurtured’ him as a composer.

6. But for every experimental musician there’s a critic nestling nearby. 

CAT/1997(Verbal Ability)

Question. 142

DIRECTIONS FOR QUESTIONS: Arrange sentences A, B, C, and D between sentences 1 and 6 to form a logical sequence of the six sentences.

1. India, which has two out of every five TB patients in the world, is on the brink of a major public health disaster. 

A. If untreated, a TB patient can die within five years.

B. Unlike AIDS, the great curse of modern sexuality, the TB germ is air-borne, which means there are no barriers to its spread.

C. The dreaded infection ranks fourth among major killers worldwide.

D. Every minute, a patient falls prey to the infection in India, which means that over five lakh people die of the disease annually.

6. Anyone, anywhere can be affected by this disease. 

CAT/1996(Verbal Ability)

Question. 143

In the following question, the answer choices suggest alternative arrangements of four or more sentences (denoted by A, B, C, D, E, F). Choose the alternative which suggests a coherent paragraph.

A. However, the severed head could not grow back if fire could be applied at once to the amputated part.

B. To get rid of this monstrosity was truly a Herculean task, for as soon as one head was cut off, two new ones replaced it.

C. Hercules accomplished the labour through the aid of an assistant who cauterized the necks as fast as Hercules cut off the heads!

D. One of the Twelve labours of Hercules was the killing of Hydra, a water monster with nine heads. 

CAT/1996(Verbal Ability)

Question. 144

In the following question, the answer choices suggest alternative arrangements of four or more sentences (denoted by A, B, C, D, E, F). Choose the alternative which suggests a coherent paragraph.

A. Still, Sophie might need open-heart surgery later in life and now be more prone to respiratory infections.

B. But with the news that his infant daughter Sophie has a hole in her heart, he appears quite vulnerable.

C. While the condition sounds bad, it is not life-threatening, and frequently corrects itself.

D. Sylvester Stallone has made millions and built a thriving career out of looking invincible.

CAT/1996(Verbal Ability)

Question. 145

In the following question, the answer choices suggest alternative arrangements of four or more sentences (denoted by A, B, C, D, E, F). Choose the alternative which suggests a coherent paragraph.

A. That Hollywood is a man’s world is certainly true, but it is not the whole truth.

B. Even Renaissance film woman, Jodie Foster, who hosts this compendium of movie history, confesses surprise at this.

C. She says that she had no idea that women were so active in the industry even in those days.

D. During the silent era, for example, female screenwriters outnumbered males 10 to 1. 

CAT/1996(Verbal Ability)

Question. 146

In the following question, the answer choices suggest alternative arrangements of four or more sentences (denoted by A, B, C, D, E, F). Choose the alternative which suggests a coherent paragraph.

A. The Saheli Program, run by the US Cross-Cultural Solutions, is offering a three-week tour of India that involves more than frenzied sightseeing.

B. Participants interested in women’s issues will learn about arranged marriages, dowries, and infanticide.

C. Holiday packages include all sorts of topics, but female infanticide must be a first for tourism.

D. Interspersed with these talks and meetings are visits to cities like New Delhi and Agra, home to the Taj Mahal.

CAT/1996(Verbal Ability)

Question. 147

In the following question, the answer choices suggest alternative arrangements of four or more sentences (denoted by A, B, C, D, E, F). Choose the alternative which suggests a coherent paragraph.

A. Its business decisions are made on the timely and accurate flow of information.

B. It has 1700 employees in 13 branches and representative offices across the Asia-Pacific region.

C. For employees to maintain a competitive edge in a fast-moving field, they must have quick access to JP Morgan’s proprietary trade-related data.

D. J P Morgan is one of the largest banking institutions in the US and a premier international trading firm.

CAT/1996(Verbal Ability)

Question. 148

In the following question, the answer choices suggest alternative arrangements of four or more sentences (denoted by A, B, C, D, E, F). Choose the alternative which suggests a coherent paragraph.

A. Something magical is happening to our planet.

B. Some are calling it a paradigm shift.

C. It’s getting smaller.

D. Others call it business transformation.

CAT/1996(Verbal Ability)

Question. 149

DIRECTIONS FOR QUESTIONS: Arrange sentences A, B, C, and D between sentences 1 and 6 to form a logical sequence of the six sentences.

1. A few years ago, hostility towards Japanese - Americans was so strong that I thought they were going to reopen the detention camps here in California. 

Today Asians are a success story.

B I cannot help making a comparison to the anti-Jewish sentiment in Nazi Germany when Jewish people were successful in business.

C But do people applaud President Clinton for improving foreign trade with Asia?

D Now, talk about the “Arkansas-Asia Connection” is broadening that hatred to include all Asian-Americans.

6. No, blinded by jealousy, they complain that it is the Asian-Americans who are reaping the wealth. (a) (b) (c) (d) 

CAT/1996(Verbal Ability)

Question. 150

DIRECTIONS FOR QUESTIONS: Arrange sentences A, B, C, and D between sentences 1 and 6 to form a logical sequence of the six sentences.

1. It doesn’t take a highly esteemed medical expert to conclude that women handle pain better than men.

A. First the men would give birth, and then take six months to recover.

B. As for labour pains, the human species would become extinct if men had to give birth.

C. They do, however, make life hell for everyone else with their non-stop complaining about how bad they feel.

D. The men in my life, including my husband and my father, would not take a Tylenol for pain if their lives depended on it.

6. And by the time they finish sharing their excruciating experience with their buddies, all reproduction would come to a halt. 

CAT/1996(Verbal Ability)

Question. 151

DIRECTIONS FOR QUESTIONS: Arrange sentences A, B, C, and D between sentences 1 and 6 to form a logical sequence of the six sentences.

1. Michael Jackson, clearly no admirer of long engagements, got married abruptly for the second time in three years. 

A. The latest wedding took place in a secret midnight ceremony in Sydney, Australia.

B. It is also the second marriage for the new missus, about whom little is known.

C. The wedding was attended by the groom’s entourage and staff, according to Jackson’s publicist.

D. The bride, 37 year old Debbie Rowe, who is carrying Jackson’s baby, wore white.

6. All that is known is that she is a nurse for Jackson’s dermatologist. 

 

CAT/1996(Verbal Ability)

Question. 152

DIRECTIONS FOR QUESTIONS: Arrange sentences A, B, C, and D between sentences 1 and 6 to form a logical sequence of the six sentences.

1. Liz Taylor isn’t just unlucky in love 

A. She, and husband Larry Fortensky, will have to pay the tab-$432,600 in court costs.

B. The duo claimed that a 1993 story about a property dispute damaged their reputations.

C. Taylor has lost a defamation suit against the National Enquirer.

D. She is unlucky in law too.

6. Alas, all levels of the California court system disagreed. 

CAT/1996(Verbal Ability)

Question. 153

DIRECTIONS FOR QUESTIONS: Arrange sentences A, B, C, and D between sentences 1 and 6 to form a logical sequence of the six sentences.

1. Since its birth, rock has produced a long string of guitar heroes. 

A. It is a list that would begin with Chuck Berry, continue on through Hendrix, page and Clapton.

B. These are musicians celebrated for their sheer instrumental talent, and their flair for expansive, showy and sometimes selfindulgent solos.

C. It would also include players of more recent vintage, like Van Halen and Living Colour’s Vernon Reid.

D. But with the advent of alternative rock and grunge, guitar heroism became uncool.

6. Guitarists like Peter Buck and Kurt Cobain shy away from exhibitionism. 

CAT/1996(Verbal Ability)

Question. 154

DIRECTIONS FOR QUESTIONS: Arrange sentences A, B, C, and D between sentences 1 and 6 to form a logical sequence of the six sentences.

1. Hiss was serving as head of the Endowment on Aug 3, 1948, when Whittaker Chambers reluctantly appeared before the House Un-American Activities Committee. 

A. Chambers, a portly rumpled man with a melodramatic style, had been a Communist courier but had broken with the party in 1938.

B. When Nixon, arranged a meeting of the two men in New York, Chambers repeated his charges and Hiss his denials.

C. Summoned as a witness, Hiss denied he had ever been a Communist or had known Chambers.

D. He told the Committee that among the members of a secret Communist cell in Washington during the 30s was Hiss.

6. Then, bizarrely, Hiss asked Chambers to open his mouth. 

CAT/1996(Verbal Ability)

Question. 155

DIRECTIONS FOR QUESTIONS: Arrange sentences A, B, C, and D between sentences 1 and 6 to form a logical sequence of the six sentences.

1. For many scientists, oceans are the cradle of life. 

A. But all over the world, chemical products and nuclear waste continue to be dumped into them.

B. Coral reefs, which are known as the most beautiful places of the submarine world, are fast disappearing.

C. The result is that many species of fish die because of this pollution.

D. Of course Man is the main cause of these problems.

6. Man has long since ruined the places he goes to -- continents and oceans alike.

CAT/1996(Verbal Ability)

Question. 156

DIRECTIONS FOR QUESTIONS: Arrange sentences A, B, C, and D between sentences 1 and 6 to form a logical sequence of the six sentences.

1. Am I one of the people who are worried that Bill Clinton’s second term might be destroyed by a constitutional crisis? 

A. On the other hand, ordinary citizens have put the campaign behind them.

B. In other words, what worries me is that Bill Clinton could exhibit a version of what George Bush used to refer to as Big Mo.

C. That is, he might have so much campaign momentum that he may not be able to stop campaigning.

D. Well, it’s true that I’ve been wondering whether a President could be impeached for refusing to stop talking about the bridge we need to build to the 21st century.

6. They prefer now to watch their favourite soaps and ads on TV rather than senators 

CAT/1996(Verbal Ability)

Question. 157

DIRECTIONS FOR QUESTIONS: Arrange sentences A, B, C, and D between sentences 1 and 6 to form a logical sequence of the six sentences.

1. Historically, stained glass was almost entirely reserved for ecclesiastical spaces 

A. By all counts, he has accomplished that mission with unmistakable style.

B. “It is my mission to bring it kicking and screaming out of that milieu,” says Clarke.

C. The first was the jewel-like windows he designed for a Cistercian Church in Switzerland

D. Two recent projects show his genius in the separate worlds of the sacred and the mundane.

6. The other was a spectacular, huge skylight in a shopping complex in Brazil. 

CAT/1996(Verbal Ability)

Question. 158

DIRECTIONS FOR QUESTIONS: Arrange sentences A, B, C, and D between sentences 1 and 6 to form a logical sequence of the six sentences.

1. So how big is the potential market? 

A. But they end up spending thousands more each year on hardware upgrades and software overhauls.

B. Analysts say the new machines will appeal primarily to corporate users.

C. An individual buyer can pick up a desktop computer for less than $2000 in America

D. For them, the NC’s best drawing card is its promise of much lower maintainence costs.

6. NC’s, which automatically load the latest version of whatever software they need, could put an end to all that.

CAT/1995(Verbal Ability)

Question. 159

In the following question, the answer choices suggest alternative arrangements of four or more sentences (denoted by A, B, C, D, E, F). Choose the alternative which suggests a coherent paragraph.

A. No other document gives us so intimate a sense of the tone and temper of the first generation English poets

B. Part of the interest of the journal is of course historical.

C. And the clues to Wordsworth’s creative processes which the journal affords are of decisive significance.

D. Not even in their own letters do Wordsworth and Coleridge stand so present before us than they do through the references in the journal

CAT/1995(Verbal Ability)

Question. 160

In the following question, the answer choices suggest alternative arrangements of four or more sentences (denoted by A, B, C, D, E, F). Choose the alternative which suggests a coherent paragraph.

A. We can never leave off wondering how that which has ever been should cease to be.

B. As we advance in life, we acquire a keener sense of the value of time.

C. Nothing else, indeed, seems of any consequence; and we become misers in this sense.

D. We try to arrest its few tottering steps and to make it linger on the brink of the grave.

CAT/1995(Verbal Ability)

Question. 161

In the following question, the answer choices suggest alternative arrangements of four or more sentences (denoted by A, B, C, D, E, F). Choose the alternative which suggests a coherent paragraph.

A. These high plans died, slowly but definitively, and were replaced by the dream of huge work on the philosophy

B. In doing whatever little he could of the new plan, the poet managed to write speculations on theology and political theory.

C. The poet’s large ambitions included the writing of a philosophic epic of the origin of evil.

D. However not much has done in this regard either, with only fragments being written.

CAT/1995(Verbal Ability)

Question. 162

In the following question, the answer choices suggest alternative arrangements of four or more sentences (denoted by A, B, C, D, E, F). Choose the alternative which suggests a coherent paragraph.

A. There is no complete knowledge about anything.

B. Our thinking is the outcome of knowledge, and knowledge is always limited.

C. Knowledge always goes hand in hand with ignorance.

D. Therefore, our thinking which is born out of knowledge, is limited under all circumstances.

CAT/1995(Verbal Ability)

Question. 163

In the following question, the answer choices suggest alternative arrangements of four or more sentences (denoted by A, B, C, D, E, F). Choose the alternative which suggests a coherent paragraph.

A. Where there is division there must be conflict not only division between man and woman, but also division as racial, religious and linguistic.

B. We said the present condition of racial divisions, linguistic divisions have brought on so many wars.

C. Also we went into the question of why does this conflict between man and man exists.

D. May we continue with what we were talking about last evening?

CAT/1995(Verbal Ability)

Question. 164

Direction for Questions: Arrange sentences A, B, C, and D between sentences 1 and 6 to form a logical sequence of the six sentences.

1. But the vessel kept going away.

A. He looked anxiously around.

B. There was nothing to see but the water and empty sky.

C. He could now barely see her funnel and masts when heaved up on a high wave.

D. He did not know for what.

6. A breaking wave slapped him in the face, choking him. 

CAT/1995(Verbal Ability)

Question. 165

Direction for Questions: Arrange sentences A, B, C, and D between sentences 1 and 6 to form a logical sequence of the six sentences.

1. All human beings are aware of the existence of a power greater than that of the mortals- the name was given to such a power by individuals is an outcome of birth, education, and choice.

A. This power provides an anchor in times of adversity, difficulty, and trouble.

B. Industrial organizations also contribute to the veneration of this power by participating in activities such as religious ceremonies and festivities organized by the employees.

C. Their other philanthropic contributions include the construction and maintenance of religious places such as temples or gurudwaras.

D. Logically, therefore such a power should be remembered in good times also.

6. The top management/ managers should participate in all such events, irrespective of theirs. 

CAT/1995(Verbal Ability)

Question. 166

Direction for Questions: Arrange sentences A, B, C, and D between sentences 1 and 6 to form a logical sequence of the six sentences.

1. Total forgiveness for a mistake committed generates a sense of complacency towards target achievement among the employees.

A. In such a situation the work ethos gets distorted and individuals get a feeling that they can get away with any lapse.

B. The feeling that they develop is ‘whether I produce results or not, the management will not punish me or does not have the guts to punish me.’

C. Also excess laxity damages management credibility because, for a long time, the management has maintained that dysfunctional behavior will result in punishment, and when something goes wrong, it fails to take specific punishment, and when something goes wrong, it fails to take specific punitive action.

D. The severity of the punishment may be reduced, by modifying it, but some action must be taken against the guilty so as to serve as a reminder for all others in the organization.

6. Moreover, it helps establish the management’s image of being firm, fair, and yet human.  

CAT/1995(Verbal Ability)

Question. 167

Direction for Questions: Arrange sentences A, B, C, and D between sentences 1 and 6 to form a logical sequence of the six sentences.

1. Currency movements can have a dramatic impact on equity returns for foreign investors.

A. This is not surprising as many developing economies try to peg their exchange rates to the US dollar or to a basket of currencies.

B. Many developing economies manage to keep exchange rate volatility lower than that in the industrial economies.

C. India has also gone in for the full float on the current account and abolished the managed exchange rate.

D. Dramatic exceptions are Argentina, Brazil, and Nigeria.

6. Another emerging market-specific risk is liquidity risk.

CAT/1995(Verbal Ability)

Question. 168

Direction for Questions: Arrange sentences A, B, C, and D between sentences 1 and 6 to form a logical sequence of the six sentences.

1. Managers must lead by example; they should not be averse to giving a hand in manual work if required.

A. They should also update their competence to guide their subordinates; this would be possible only if they keep in regular touch with new processes, machines, instruments, gauges, systems, and gadgets.

B. Work must be allocated to different groups and team members in clear, specific terms.

C. Too much of wall -building is detrimental to the exercise of the ‘personal charisma’ of the leader whose presence should not be felt only through notice, circulars, or memos, but by being seen physically.

D. Simple, clean living among one’s people should be insisted upon.

6. This would mean the maintaining of an updated organization chart; laying down job descriptions; identifying key result areas; setting personal targets; and above all, monitoring performance, to meet organizational goals.  

CAT/1995(Verbal Ability)

Question. 169

Direction for Questions: Arrange sentences A, B, C, and D between sentences 1 and 6 to form a logical sequence of the six sentences.

1. The top management should perceive the true worth of people and only then make friends. 

A. Such ‘true friends’ are very few and very rare.

B. Factors such as affluence, riches, outward sophistication, and conceptual abilities are not prerequisites for genuine friendship.

C. Such people must be respected and kept close to the heart.

D. Business realities call for developing a large circle of acquaintances and contacts: however, all of them will be motivated by their own self-interest and it would be wrong to treat them as genuine friends.

6. There is always a need for real friends to whom one can turn for balanced, unselfish advice, more so when one is caught in a dilemma. 

CAT/1995(Verbal Ability)

Question. 170

Direction for Questions: Arrange sentences A, B, C, and D between sentences 1 and 6 to form a logical sequence of the six sentences.

1. Conflicting demands for resources are always voiced by different functions/ departments in an organization.

A. Every manager examines the task entrusted to him and evaluates the resources required.

B. Availability of resources in full measure makes task achievement easy because it reduces the effort needed to somewhat make-do.

C. A safety cushion is built into demand for resources, to offset the adverse impact of any cut imposed by the seniors.

D. This aspect needs to be understood as reality.

6. Dynamic, energetic, growth-oriented, and wise management are always confronted with the inadequacy of resources with respect to one some of the four M’s (men, machines, money, and materials ) and the two T’s (time and technology). 

CAT/1995(Verbal Ability)

Question. 171

Direction for Questions: Arrange sentences A, B, C, and D between sentences 1 and 6 to form a logical sequence of the six sentences.

 

1. Managers especially the successful ones, should guard against ascribing to themselves qualities and attributes which they may not have, or may have in a measure much less than what they think they have.

A. External appearances can be deceptive.

B. To initiate action, without being in possession of full facts, can lead to disastrous results.

C. Also, one should develop confidants who can be used as sounding boards, in order to check one’s own thinking against that of others.

D. It is also useful to be open to receive feedback about oneself so that a real understanding of the ‘self’ exists.

6. A false perception can be likened to wearing coloured glasses - all facts get tainted by colour of the glass and the mind interprets them wrongly to fit into the perception. 

CAT/1995(Verbal Ability)

Question. 172

Direction for Questions: Arrange sentences A, B, C, and D between sentences 1 and 6 to form a logical sequence of the six sentences.

1. Mangers need to differentiate among those who commit an error once, those who are repetitively arrant but can be corrected, and those who are basically wicked.

A. The persons in this category will resort to sweet-talk and make all sorts of promises on being caught, but at the first opportunity will revert to their bad ways.

B. Managers must take ruthless action against the basically wicked and ensure their separation from the organization at the earliest.

C. The first category needs to be corrected softly and duly counselled; the second category should be dealt with firmly and duly counselled till they realize the danger of persisting with their errant behaviour.

D. It is the last category of whom the managers must be most wary.

6. The punishment must be fair and based on the philosophy of giving all the possible opportunities and help prior to taking ruthless action. 

CAT/1995(Verbal Ability)

Question. 173

DIRECTIONS FOR QUESTIONS: Arrange sentences A, B, C, and D between sentences 1 and 6 to form a logical sequence of the six sentences.

1. Despite the passage of time, a large number of conflicts continue to remain alive, because the wronged parties, in reality, or in imagination, wish to take revenge upon each other, thus creating a vicious circle. 

A. At times, managers are called upon to take ruthless decisions in the long-term interests of the organization.

B. People hurt others, at times knowingly, to teach them a lesson and, at other times, because they lack correct understanding of the other person’s stand.

C. The delegation of any power, to any person, is never absolute.

D. Every ruthless decision will be easier to accept if the situation at the moment of committing the act is objectively analyzed, shared openly, and discussed rationally.

6. power is misused, its effects can last only for a while since employees are bound to confront it someday, more so, the talented ones. 

CAT/1994(Verbal Ability)

Question. 174

In the following question, the answer choices suggest alternative arrangements of four or more sentences (denoted by A, B, C, D, E, F). Choose the alternative which suggests a coherent paragraph.

A. To have settled one’s affairs is very good preparation to leading the rest of one’s life without concern for the future.

B. When I have finished this book I shall know where I stand.

C. One does not die immediately after one has made one’s will; one make’s one’s will as a precaution.

D. I can afford then to do what I choose with the years that remain to me. 

CAT/1994(Verbal Ability)

Question. 175

In the following question, the answer choices suggest alternative arrangements of four or more sentences (denoted by A, B, C, D, E, F). Choose the alternative which suggests a coherent paragraph.

A. It is said that India has always been in a hurry to conform to western thought, especially the American

B. Even the smaller countries have the guts to take a firm contrarian stand if they feel the policies happen to compromise their country’s interest.

C. It's one thing to sprout theories on liberalization, and entirely another to barter the interests of the nation in its name.

D. In this case too, while a large number of countries are yet to ratify the GATT, India has not only ratified the treaty but is also preparing to amend the Patents Act.

CAT/1994(Verbal Ability)

Question. 176

In the following question, the answer choices suggest alternative arrangements of four or more sentences (denoted by A, B, C, D, E, F). Choose the alternative which suggests a coherent paragraph.

A. But instead you are faced with another huge crag and the weary trail continues

B. No, the path winds on, and another mountain bars your way.

C. When for days you have been going through a mountain pass, a moment comes when you are sure that after winding around the great mass of rock in front of you, you will come upon the plain.

D. Surely after this you will see the plain.

CAT/1994(Verbal Ability)

Question. 177

In the following question, the answer choices suggest alternative arrangements of four or more sentences (denoted by A, B, C, D, E, F). Choose the alternative which suggests a coherent paragraph.

A. During one exhibition, however, some air became mixed with the hydrogen, and in the words of the shaken performer: “The explosion was so dreadful that I imagined all my teeth had been blown out!”

B. An entertainer would finish his acts by blowing the hydrogen he had inhaled towards a lighted candle; as the hydrogen caught fire, flames would shoot menacingly from his lips.

C. A paper bag filled with hydrogen amazed guests by zooming off into space.

D. When people learned about its unique lighter-than-air property, they began to use it in all sorts of parlor stunts.

CAT/1994(Verbal Ability)

Question. 178

In the following question, the answer choices suggest alternative arrangements of four or more sentences (denoted by A, B, C, D, E, F). Choose the alternative which suggests a coherent paragraph.

A. It is exciting and various

B. I am a writer as I might have been a doctor or a lawyer.

C. The writer is free to work in what he believes.

D. It is so pleasant a profession that it is not surprising if a vast number of persons adopt it who have no qualifications for it.

CAT/1994(Verbal Ability)

Question. 179

In the following question, the answer choices suggest alternative arrangements of four or more sentences (denoted by A, B, C, D, E, F). Choose the alternative which suggests a coherent paragraph.

A. there was the hope that in another existence greater happiness would reward one.

B. previous existence, and the effort to do better would be less difficult too when

C. it would be less difficult to bear the evils of one’s own life if

D. one could think that they were but the necessary outcome of one’s errors in a

CAT/1994(Verbal Ability)

Question. 180

In the following question, the answer choices suggest alternative arrangements of four or more sentences (denoted by A, B, C, D, E, F). Choose the alternative which suggests a coherent paragraph.

A. he can only renew himself if his soul

B. he renews himself and

C. the writer can only be fertile if

D. is constantly enriched by fresh experience 

CAT/1994(Verbal Ability)

Question. 181

In the following question, the answer choices suggest alternative arrangements of four or more sentences (denoted by A, B, C, D, E, F). Choose the alternative which suggests a coherent paragraph.

A. But a masterpiece is

B. Untaught genius

C. A laborious career than as the lucky fluke of

D. More likely to come as the culminating point of 

CAT/1994(Verbal Ability)

Question. 182

In the following question, the answer choices suggest alternative arrangements of four or more sentences (denoted by A, B, C, D, E, F). Choose the alternative which suggests a coherent paragraph.

A. What interests you is the way in which you have created the illusion

B. They are angry with you, for it was

C. The public is easily disillusioned and then

D. The illusion they loved; they do not understand that

CAT/1994(Verbal Ability)

Question. 183

In the following question, the answer choices suggest alternative arrangements of four or more sentences (denoted by A, B, C, D, E, F). Choose the alternative which suggests a coherent paragraph.

A. An adequate physical and social infrastructure level

B. The pattern of spatial growth in these towns as also to

C. The failure of the government to ensure

D. The roots of the riots are related to 

CAT/1994(Verbal Ability)

Question. 184

Directions for Question: Arrange sentences A, B, C, and D between sentences 1 and 6 to form a logical sequence of the six sentences.

1. It is often said that good actors can get out of play more than the author has put into it.

A. A good actor, bringing to a part his own talent, often gives it a value that the layman on reading the play had not seen in it, but at the utmost, he can do no more than reaching the ideal that the author has seen in his mind’s eye.

B. In all my plays I have been fortunate enough to have some of the parts acted as I wanted, but in none have I had all the parts so acted.

C. That is not true.

D. He has to be an actor of address to do this; for the most part, the author has to be satisfied with an approximation of the performance he visualized.

6. is so obviously inevitable, for the actor who is suited to a certain role may very well be engaged and you have to put up with the second or third best because there is no help for it.

CAT/1994(Verbal Ability)

Question. 185

Directions for Question: Arrange sentences A, B, C, and D between sentences 1 and 6 to form a logical sequence of the six sentences.

1. I can think of no serious prose play that has survived the generation that gave it birth.

A. They are museum pieces.

B. They are revived now and then because a famous part tempts a leading actor, or a manager in want of a stop-gap thinks he will put on a play on which he has no royalties to pay.

C. A few comedies have haphazardly traveled down a couple of centuries or so.

D. The audience laughs at their wit with politeness and at their farce with embarrassment.

6. They are not held nor taken out of themselves. 

CAT/1994(Verbal Ability)

Question. 186

Directions for Question: Arrange sentences A, B, C, and D between sentences 1 and 6 to form a logical sequence of the six sentences.

1. The wind had savage allies. 

A. If it had not been for my closely fitted helmet, the explosions might have shattered my eardrums.

B. The first clap of thunder came as a deafening explosion that literally shook my teeth.

C. I didn’t hear the thunder, I actually felt it - an almost unbearable physical experience.

D. I saw lighting all around me in every shape imaginable.

6. When very close, it raining so torrentially that I thought I would drown in mid-air. 

CAT/1994(Verbal Ability)

Question. 187

Directions for Question: Arrange sentences A, B, C, and D between sentences 1 and 6 to form a logical sequence of the six sentences.

1. All human beings are aware of the existence of a power greater than that of the mortals - the name was given to such a power by individuals is an outcome of birth, education, and choice.

A. Logically, therefore such a power should be remembered in good times also.

B. Their other philanthropic contributions include the construction and maintenance of religious places such as temples or gurudwaras.

C. Industrial organizations also contribute to the veneration of this power by participating in activities such as religious ceremonies and festivities organized by the employees.

D. This power provides an anchor in times of adversity, difficulty, and trouble. 6. The top management/managers should participate in all such events, irrespective of their personal choice. 

CAT/1994(Verbal Ability)

Question. 188

Direction for Questions: Arrange sentences A, B, C, and D between sentences 1 and 6 to form a logical sequence of the six sentences.

1. A thorough knowledge of the path or course to be followed is essential for achieving success.

A. Seniors must show the path clearly by laying down the precise expectations of the management in terms of the job description, key result areas, and personal targets.

B. They should also ‘light the path’ by personal example.

C. Advice tendered or help offered must be objectively evaluated for its effectiveness in achieving the desired goal.

D. A display of arrogance and a false sense of ‘self-worth’, in order to belittle those who come to help prove dysfunctional.

6. The individuality of each employee must be respected.